Нормальная реакция опоры формула: Что такое сила нормальной реакции? (статья)

Содержание

Сила нормальной реакции опоры. Формулы по физике. Задача с доской, двумя опорами и грузом

Инструкция

Случай 1. Формула для скольжения: Fтр = мN, где м – коэффициент трения скольжения, N – сила реакции опоры, Н. Для тела, скользящего по горизонтальной плоскости, N = G = mg, где G — вес тела, Н; m – масса тела, кг; g – ускорение свободного падения, м/с2. Значения безразмерного коэффициента м для данной пары материалов даны в справочной . Зная массу тела и пару материалов. скользящих друг относительно друга, найдите силу трения.

Случай 2. Рассмотрите тело, скользящее по горизонтальной поверхности и двигающееся равноускоренно. На него действуют четыре силы: сила, приводящее тело в движение, сила тяжести, сила реакции опоры, сила трения скольжения. Так как поверхность горизонтальная, сила реакции опоры и сила тяжести направлены вдоль одной прямой и уравновешивают друг друга. Перемещение описывает уравнение: Fдв — Fтр = ma; где Fдв – модуль силы, приводящей тело в движение, Н; Fтр – модуль силы трения, Н; m – масса тела, кг; a – ускорение, м/с2.

2 = 0,8 м/с2. Теперь найдите силу трения: Fтр = ma = 0,8*1 = 0,8 Н.

Случай 4. На тело, самопроизвольно скользящее по наклонной плоскости, действуют три силы: сила тяжести (G), сила реакции опоры (N) и сила трения (Fтр). Сила тяжести может быть записана в таком виде: G = mg, Н, где m – масса тела, кг; g – ускорение свободного падения, м/с2. Поскольку эти силы направлены не вдоль одной прямой, запишите уравнение движения в векторном виде.

Сложив по правилу параллелограмма силы N и mg, вы получите результирующую силу F’. Из рисунка можно сделать выводы: N = mg*cosα; F’ = mg*sinα. Где α – угол наклона плоскости. Силу трения можно записать формулой: Fтр = м*N = м*mg*cosα. Уравнение для движения принимает вид: F’-Fтр = ma. Или: Fтр = mg*sinα-ma.

Случай 5. Если же к телу приложена дополнительная сила F, направленная вдоль наклонной плоскости, то сила трения будет выражаться: Fтр = mg*sinα+F-ma, если направление движения и силы F совпадают. Или: Fтр = mg*sinα-F-ma, если сила F противодействует движению.

2 = 0,8 м/с2. Вычислите силу трения в первом случае: Fтр = 1*9,8*sin(45о)-1*0,8 = 7,53 Н. Определите силу трения во втором случае: Fтр = 1*9,8*sin(45о)+2-1*0,8= 9,53 Н.

Случай 6. Тело двигается по наклонной поверхности равномерно. Значит, по второму закону Ньютона система находится в равновесии. Если скольжение самопроизвольное, движение тела подчиняется уравнению: mg*sinα = Fтр.

Если же к телу приложена дополнительная сила (F), препятствующая равноускоренному перемещению, выражение для движения имеет вид: mg*sinα–Fтр-F = 0. Отсюда найдите силу трения: Fтр = mg*sinα-F.

Источники:

  • скольжение формула

Коэффициент трения – это совокупность характеристик двух тел, которые соприкасаются друг с другом. Существует несколько видов трения: трение покоя, трение скольжения и трение качения. Трение покоя представляет собой трение тело, которое находилось в покое, и было приведено в движение. Трение скольжения происходит при движении тела, данное трение меньше трения покоя.

А трение качения происходит, когда тело катиться по поверхности. Обозначается трение в зависимости от вида, следующим образом: μск — трение скольжения, μо- трение покоя, μкач – трение качения.

Инструкция

При определении коэффициента трения в ходе эксперимента, тело размещается на плоскости под наклоном и вычисляется угол наклона. При этом учитывать, что при определении коэффициента трения покоя заданное тело двигаться, а при определении коэффициента трения скольжения движется со скоростью, которая постоянна.

Коэффициент трения можно также вычислить в ходе эксперимента. Необходимо поместить объект на наклонную плоскость и вычислить угол наклона. Таким образом, коэффициент трения определяется по формуле: μ=tg(α), где μ — сила трения, α – угол наклона плоскости.

Видео по теме

При относительном движении двух тел между ними возникает трение. Оно также может возникнуть при движении в газообразной или жидкой среде. Трение может как мешать, так и способствовать нормальному движению. В результате этого явления на взаимодействующие тела действует сила трения .

Инструкция

Наиболее общий случай рассматривает силу , когда одно из тел закреплено и покоится, а другое скользит по его поверхности. Со стороны тела, по которому скользит движущееся тело, на последнее действует сила реакции опоры, направленная перпендикулярно плоскости скольжения. Эта сила буквой N.Тело может также и покоится относительно закрепленного тела. Тогда сила трения, действующая на него Fтр

В случае движения тела относительно поверхности закрепленного тела сила трения скольжения становится равна произведения коэффициента трения на силу реакции опоры: Fтр = ?N.

Пусть теперь на тело действует постоянная сила F>Fтр = ?N, параллельная поверхности соприкасающихся тел. При скольжении тела, результирующая составляющая силы в горизонтальном направлении будет равна F-Fтр. Тогда по второму закону Ньютона, ускорение тела будет связано с результирующей силой по формуле: a = (F-Fтр)/m. Отсюда, Fтр = F-ma. Ускорение тела можно найти из кинематических соображений.

Часто рассматриваемый частный случай силы трения проявляется при соскальзывании тела с закрепленной наклонной плоскости. Пусть? — угол наклона плоскости и пусть тело соскальзывает равномерно, то есть без ускорения. Тогда уравнения движения тела будут выглядеть так: N = mg*cos?, mg*sin? = Fтр = ?N. Тогда из первого уравнения движения силу трения можно выразить как Fтр = ?mg*cos?.Если тело движется по наклонной плоскости с ускорением a, то второе уравнение движение будет иметь вид: mg*sin?-Fтр = ma. Тогда Fтр = mg*sin?-ma.

Видео по теме

Если сила, направленная параллельно поверхности, на которой стоит тело, превышает силу трения покоя, то начнется движение. Оно будет продолжаться до тех пор, пока движущая сила будет превышать силу трения скольжения, зависящую от коэффициента трения. Рассчитать этот коэффициент можно самостоятельно.

Вам понадобится

  • Динамометр, весы, транспортир или угломер

Инструкция

Найдите массу тела в килограммах и установите его на ровную поверхность. Присоедините к нему динамометр, и начинайте двигать тело. Делайте это таким образом, чтобы показатели динамометра стабилизировались, поддерживая постоянную скорость . В этом случае сила тяги, измеренная динамометром, будет равна с одной стороны силе тяги, которую показывает динамометр, а с другой стороны силе , умноженной на скольжения.

Сделанные измерения позволят найти данный коэффициент из уравнения. Для этого поделите силу тяги на массу тела и число 9,81 (ускорение свободного падения) μ=F/(m g). Полученный коэффициент будет один и тот же для всех поверхностей такого же типа, как и те на которых производилось измерение. Например, если тело из двигалось по деревянной доске, то этот результат будет справедлив для всех деревянных тел, двигающихся скольжением по дереву, с учетом качества его обработки (если поверхности шершавые, значение коэффициента трения скольжения измениться).

Можно измерить коэффициент трения скольжения и другим способом. Для этого установите тело на плоскости, которая может менять свой угол относительно горизонта. Это может быть обыкновенная дощечка. Затем начинайте аккуратно поднимать ее за один край. В тот момент, когда тело придет в движение, скатываясь в плоскости как сани с горки, найдите угол ее уклона относительно горизонта. Важно, чтобы тело при этом не двигалось с ускорением. В этом случае, измеренный угол будет предельно малым, при котором тело начнет двигаться под действием силы тяжести. Коэффициент трения скольжения будет равен тангенсу этого угла μ=tg(α).

Сила нормальной реакции — сила, действующая на тело со стороны опоры (или подвеса). При соприкосновении тел вектор силы реакции направлен перпендикулярно поверхности соприкосновения. Для расчёта используется следующая формула:

|\vec N|= mg \cos \theta,

где |\vec N| — модуль вектора силы нормальной реакции, m — масса тела, g — ускорение свободного падения , \theta — угол между плоскостью опоры и горизонтальной плоскостью.

Согласно третьему закону Ньютона , модуль силы нормальной реакции |\vec N| равен модулю веса тела |\vec P|, но их вектора — коллинеарные противоположно направленные:

\vec N= -\vec P.

Из закона Амонтона — Кулона следует, что для модуля вектора силы нормальной реакции справедливо соотношение:

|\vec N|= \frac{|\vec F|}{k},

где \vec F — сила трения скольжения , а k — коэффициент трения.

Поскольку сила трения покоя вычисляется по формуле

|\vec f|= mg \sin \theta,

то мы можем экспериментальным путём найти такое значение угла \theta, при котором сила трения покоя будет равна силе трения скольжения:

mg \sin \theta = k mg \cos \theta.

Отсюда выразим коэффициент трения:

k = \mathrm{tg}\ \theta.

Напишите отзыв о статье «Сила нормальной реакции»

Отрывок, характеризующий Сила нормальной реакции

Все историки согласны в том, что внешняя деятельность государств и народов, в их столкновениях между собой, выражается войнами; что непосредственно, вследствие больших или меньших успехов военных, увеличивается или уменьшается политическая сила государств и народов.
Как ни странны исторические описания того, как какой нибудь король или император, поссорившись с другим императором или королем, собрал войско, сразился с войском врага, одержал победу, убил три, пять, десять тысяч человек и вследствие того покорил государство и целый народ в несколько миллионов; как ни непонятно, почему поражение одной армии, одной сотой всех сил народа, заставило покориться народ, – все факты истории (насколько она нам известна) подтверждают справедливость того, что большие или меньшие успехи войска одного народа против войска другого народа суть причины или, по крайней мере, существенные признаки увеличения или уменьшения силы народов. Войско одержало победу, и тотчас же увеличились права победившего народа в ущерб побежденному. Войско понесло поражение, и тотчас же по степени поражения народ лишается прав, а при совершенном поражении своего войска совершенно покоряется.
Так было (по истории) с древнейших времен и до настоящего времени. Все войны Наполеона служат подтверждением этого правила. По степени поражения австрийских войск – Австрия лишается своих прав, и увеличиваются права и силы Франции. Победа французов под Иеной и Ауерштетом уничтожает самостоятельное существование Пруссии.

Силу действующую на тело со стороны опоры (или подвеса), называют силой реакции опоры. При соприкосновении тел сила реакции опоры направлена перпендикулярно поверхности соприкосновения. Если тело лежит на горизонтальном неподвижном столе, сила реакции опоры направлена вертикально вверх и уравновешивает силу тяжести:

Wikimedia Foundation . 2010 .

Смотреть что такое «Сила нормальной реакции опоры» в других словарях:

    Сила трения скольжения силы, возникающие между соприкасающимися телами при их относительном движении. Если между телами отсутствует жидкая или газообразная прослойка (смазка), то такое трение называется сухим. В противном случае, трение… … Википедия

    Запрос «сила» перенаправляется сюда; см. также другие значения. Сила Размерность LMT−2 Единицы измерения СИ … Википедия

    Запрос «сила» перенаправляется сюда; см. также другие значения. Сила Размерность LMT−2 Единицы измерения СИ ньютон … Википедия

    Закон Амонтона Кулона эмпирический закон, устанавливающий связь между поверхностной силой трения, возникающей при относительном скольжении тела, с силой нормальной реакции, действующей на тело со стороны поверхности. Сила трения,… … Википедия

    Силы трения скольжения силы, возникающие между соприкасающимися телами при их относительном движении. Если между телами отсутствует жидкая или газообразная прослойка (смазка), то такое трение называется сухим. В противном случае, трение… … Википедия

    Трение покоя, трение сцепления сила, возникающая между двумя контактирующими телами и препятствующая возникновению относительного движения. Эту силу необходимо преодолеть для того, чтобы привести два контактирующих тела в движение друг… … Википедия

    Сюда перенаправляется запрос «Прямохождение». На эту тему нужна отдельная статья. Ходьба человека наиболее естественная локомоция человека. Автоматизированный двигательный акт, осуществляющийся в результате сложной координированной деятельности… … Википедия

    Цикл ходьбы: опора на одну ногу двуопорный период опора на другую ногу… Ходьба человека наиболее естественная локомоция человека. Автоматизированный двигательный акт, осуществляющийся в результате сложной координированной деятельности скелетных … Википедия

    Сила трения при скольжении тела о поверхность не зависит от площади соприкосновения тела с поверхностью, но зависит от силы нормальной реакции этого тела и от состояния окружающей среды. Сила трения скольжения возникает при скольжении данного… … Википедия

    Закон Амонтона Кулона сила трения при скольжении тела о поверхность не зависит от площади соприкосновения тела с поверхностью, но зависит от силы нормальной реакции этого тела и от состояния окружающей среды. Сила трения скольжения возникает при… … Википедия

Необходимо знать точку приложения и направление каждой силы. Важно уметь определить какие именно силы действуют на тело и в каком направлении. Сила обозначается как , измеряется в Ньютонах. Для того, чтобы различать силы, их обозначают следующим образом

Ниже представлены основные силы, действующие в природе. Придумывать не существующие силы при решении задач нельзя!

Сил в природе много. Здесь рассмотрены силы, которые рассматриваются в школьном курсе физики при изучении динамики. А также упомянуты другие силы, которые будут рассмотрены в других разделах.

Сила тяжести

На каждое тело, находящееся на планете, действует гравитация Земли . Сила, с которой Земля притягивает каждое тело, определяется по формуле

Точка приложения находится в центре тяжести тела. Сила тяжести всегда направлена вертикально вниз .


Сила трения

Познакомимся с силой трения. Эта сила возникает при движении тел и соприкосновении двух поверхностей. Возникает сила в результате того, что поверхности, если рассмотреть под микроскопом, не являются гладкими, как кажутся. Определяется сила трения по формуле:

Сила приложена в точке соприкосновения двух поверхностей. Направлена в сторону противоположную движению.

Сила реакции опоры

Представим очень тяжелый предмет, лежащий на столе. Стол прогибается под тяжестью предмета. Но согласно третьему закону Ньютона стол воздействует на предмет с точно такой же силой, что и предмет на стол. Сила направлена противоположно силе, с которой предмет давит на стол. То есть вверх. Эта сила называется реакцией опоры. Название силы «говорит» реагирует опора . Эта сила возникает всегда, когда есть воздействие на опору. Природа ее возникновения на молекулярном уровне. Предмет как бы деформировал привычное положение и связи молекул (внутри стола), они, в свою очередь, стремятся вернуться в свое первоначальное состояние, «сопротивляются».

Абсолютно любое тело, даже очень легкое (например,карандаш, лежащий на столе), на микроуровне деформирует опору. Поэтому возникает реакция опоры.

Специальной формулы для нахождения этой силы нет. Обозначают ее буквой , но эта сила просто отдельный вид силы упругости, поэтому она может быть обозначена и как

Сила приложена в точке соприкосновения предмета с опорой. Направлена перпендикулярно опоре.


Так как тело представляем в виде материальной точки, силу можно изображать с центра

Сила упругости

Это сила возникает в результате деформации (изменения первоначального состояния вещества). Например, когда растягиваем пружину, мы увеличиваем расстояние между молекулами материала пружины. Когда сжимаем пружину — уменьшаем. Когда перекручиваем или сдвигаем. Во всех этих примерах возникает сила, которая препятствует деформации — сила упругости.

Закон Гука


Сила упругости направлена противоположно деформации.

Так как тело представляем в виде материальной точки, силу можно изображать с центра

При последовательном соединении, например, пружин жесткость рассчитывается по формуле

При параллельном соединении жесткость

Жесткость образца. Модуль Юнга.

Модуль Юнга характеризует упругие свойства вещества. Это постоянная величина, зависящая только от материала, его физического состояния. Характеризует способность материала сопротивляться деформации растяжения или сжатия. Значение модуля Юнга табличное.

Подробнее о свойствах твердых тел .

Вес тела

Вес тела — это сила, с которой предмет воздействует на опору. Вы скажете, так это же сила тяжести! Путаница происходит в следующем: действительно часто вес тела равен силе тяжести, но это силы совершенно разные. Сила тяжести — сила, которая возникает в результате взаимодействия с Землей. Вес — результат взаимодействия с опорой. Сила тяжести приложена в центре тяжести предмета, вес же — сила, которая приложена на опору (не на предмет)!

Формулы определения веса нет. Обозначается эта силы буквой .

Сила реакции опоры или сила упругости возникает в ответ на воздействие предмета на подвес или опору, поэтому вес тела всегда численно одинаков силе упругости, но имеет противоположное направление.

Сила реакции опоры и вес — силы одной природы, согласно 3 закону Ньютона они равны и противоположно направлены. Вес — это сила, которая действует на опору, а не на тело. Сила тяжести действует на тело.

Вес тела может быть не равен силе тяжести. Может быть как больше, так и меньше, а может быть и такое, что вес равен нулю. Это состояние называется невесомостью . Невесомость — состояние, когда предмет не взаимодействует с опорой, например, состояние полета: сила тяжести есть, а вес равен нулю!

Определить направление ускорения возможно, если определить, куда направлена равнодействующая сила

Обратите внимание, вес — сила, измеряется в Ньютонах. Как верно ответить на вопрос: «Сколько ты весишь»? Мы отвечаем 50 кг, называя не вес, а свою массу! В этом примере, наш вес равен силе тяжести, то есть примерно 500Н!

Перегрузка — отношение веса к силе тяжести

Сила Архимеда

Сила возникает в результате взаимодействия тела с жидкость (газом), при его погружении в жидкость (или газ). Эта сила выталкивает тело из воды (газа). Поэтому направлена вертикально вверх (выталкивает). Определяется по формуле:

В воздухе силой Архимеда пренебрегаем.

Если сила Архимеда равна силе тяжести, тело плавает. Если сила Архимеда больше, то оно поднимается на поверхность жидкости, если меньше — тонет.

Электрические силы

Существуют силы электрического происхождения. Возникают при наличии электрического заряда. Эти силы, такие как сила Кулона , сила Ампера , сила Лоренца , подробно рассмотрены в разделе Электричество .

Схематичное обозначение действующих на тело сил

Часто тело моделируют материальной точкой . Поэтому на схемах различные точки приложения переносят в одну точку — в центр, а тело изображают схематично кругом или прямоугольником.

Для того, чтобы верно обозначить силы, необходимо перечислить все тела, с которыми исследуемое тело взаимодействует. Определить, что происходит в результате взаимодействия с каждым: трение, деформация, притяжение или может быть отталкивание. Определить вид силы, верно обозначить направление. Внимание! Количество сил будет совпадать с числом тел, с которыми происходит взаимодействие.

Главное запомнить

1) Силы и их природа;
2) Направление сил;
3) Уметь обозначить действующие силы

Различают внешнее (сухое) и внутреннее (вязкое) трение. Внешнее трение возникает между соприкасающимися твердыми поверхностями, внутреннее — между слоями жидкости или газа при их относительном движении. Существует три вида внешнего трения: трение покоя, трение скольжения и трение качения.

Трение качения определяется по формуле

Сила сопротивления возникает при движении тела в жидкости или в газе. Величина силы сопротивления зависит от размеров и формы тела, скорости его движения и свойств жидкости или газа. При небольших скоростях движения сила сопротивления пропорциональна скорости тела

При больших скоростях пропорциональна квадрату скорости

Рассмотрим взаимное притяжение предмета и Земли. Между ними, согласно закону гравитации возникает сила

А сейчас сравним закон гравитации и силу тяжести

Величина ускорения свободного падения зависит от массы Земли и ее радиуса! Таким образом, можно высчитать, с каким ускорением будут падать предметы на Луне или на любой другой планете, используя массу и радиус той планеты.

Расстояние от центра Земли до полюсов меньше, чем до экватора. Поэтому и ускорение свободного падения на экваторе немного меньше, чем на полюсах. Вместе с тем, следует отметить, что основной причиной зависимости ускорения свободного падения от широты местности, является факт вращения Земли вокруг своей оси.

При удалении от поверхности Земли сила земного тяготения и ускорения свободного падения изменяются обратно пропорционально квадрату расстояния до центра Земли.


Положим камень на горизонтальную крышку стола, стоящего на Земле (рис. 104). Поскольку ускорение камня относительно Земли равно пулю, то по второму закону Ньютона сумма действующих на него сил равна нулю. Следовательно, действие на камень силы тяжести m · g должно компенсироваться какими-то другими силами. Ясно, что под действием камня крышка стола деформируется. Поэтому со стороны стола на камень действует сила упругости. Если считать, что камень взаимодействует лишь с Землей и крышкой стола, то сила упругости должна уравновешивать силу тяжести: F упр = -m · g. Эту силу упругости называют силой реакции опоры и обозначают латинской буквой N. Так как ускорение свободного падения направлено вертикально вниз, сила N направлена вертикально вверх – перпендикулярно поверхности крышки стола.

Поскольку крышка стола действует на камень, то по третьему закону Ньютона и камень действует на крышку стола силой P = -N (рис. 105). Эту силу называют весом .

Весом тела называют силу, с которой это тело действует на подвес или опору, находясь относительно подвеса или опоры в неподвижном состоянии.

Ясно, что в рассмотренном случае вес камня равен силе тяжести: P = m · g. Это будет верно для любого тела, покоящегося на подвесе (опоре) относительно Земли (рис. 106). Очевидно, что в этом случае точка крепления подвеса (или опора) неподвижна относительно Земли.

Для тела, покоящегося на неподвижном относительно Земли подвесе (опоре), вес тела равен силе тяжести.

Вес тела также будет равен действующей на тело силе тяжести в случае, если тело и подвес (опора) движутся относительно Земли равномерно прямолинейно.

Если же тело и подвес (опора) движутся относительно Земли с ускорением так, что тело остается неподвижным относительно подвеса (опоры), то вес тела не будет равен силе тяжести.

Рассмотрим пример. Пусть тело массой m лежит на полу лифта, ускорение a которого направлено вертикально вверх (рис. 107). Будем считать, что на тело действуют только сила тяжести m · g и сила реакции пола N. (Вес тела действует не на тело, а на опору – пол лифта.) В системе отсчета, неподвижной относительно Земли, тело на полу лифта движется вместе с лифтом с ускорением a. В соответствии со вторым законом Ньютона произведение массы тела на ускорение равно сумме всех действующих на тело сил. Поэтому: m · a = N — m · g.

Следовательно, N = m · a + m · g = m · (g + a). Значит, если лифт имеет ускорение, направленное вертикально вверх, то модуль силы N реакции пола будет больше модуля силы тяжести. В самом деле, сила реакции пола должна не только скомпенсировать действие силы тяжести, но и придать телу ускорение в положительном направлении оси X.

Сила N – это сила, с которой пол лифта действует на тело. По третьему закону Ньютона тело действует на пол с силой P, модуль которой равен модулю N, но направлена сила P в противоположную сторону. Эта сила является весом тела в движущемся лифте. Модуль этой силы P = N = m · (g + a). Таким образом, в лифте, движущемся с направленным вверх относительно Земли ускорением, модуль веса тела больше модуля силы тяжести .

Такое явление называют перегрузкой .

Например, пусть ускорение а лифта направлено вертикально вверх и его значение равно g, т. е. a = g. В этом случае модуль веса тела – силы, действующей на пол лифта, – будет равен P = m · (g + a) = m · (g + g) = 2m · g. То есть вес тела при этом будет в два раза больше, чем в лифте, который относительно Земли покоится или движется равномерно прямолинейно.

Для тела на подвесе (или опоре), движущемся с ускорением относительно Земли, направленным вертикально вверх, вес тела больше силы тяжести.

Отношение веса тела в движущемся ускоренно относительно Земли лифте к весу этого же тела в покоящемся или движущемся равномерно прямолинейно лифте называют коэффициентом перегрузки или, более кратко, перегрузкой .

Коэффициент перегрузки (перегрузка) – отношение веса тела при перегрузке к силе тяжести, действующей на тело.

В рассмотренном выше случае перегрузка равна 2. Понятно, что если бы ускорение лифта было направлено вверх и его значение было равно a = 2g, то коэффициент перегрузки был бы равен 3.

Теперь представим себе, что тело массой m лежит на полу лифта, ускорение которого a относительно Земли направлено вертикально вниз (противоположно оси X). Если модуль a ускорения лифта будет меньше модуля ускорения свободного падения, то сила реакции пола лифта по-прежнему будет направлена вверх, в положительном направлении оси X, а ее модуль будет равен N = m · (g — a). Следовательно, модуль веса тела будет равен P = N = m · (g — a), т. е. будет меньше модуля силы тяжести. Таким образом, тело будет давить на пол лифта с силой, модуль которой меньше модуля силы тяжести.

Это ощущение знакомо каждому, кто ездил на скоростном лифте или качался на больших качелях. При движении вниз из верхней точки вы чувствуете, что ваше давление на опору уменьшается. Если же ускорение опоры положительно (лифт и качели начинают подниматься), вас сильнее прижимает к опоре.

Если ускорение лифта относительно Земли будет направлено вниз и равно по модулю ускорению свободного падения (лифт свободно падает), то сила реакции пола станет равной нулю: N = m · (g — a) = m · (g — g) = 0. В этом случае пол лифта перестанет давить на лежащее на нем тело. Следовательно, согласно третьему закону Ньютона и тело не будет давить на пол лифта, совершая вместе с лифтом свободное падение. Вес тела станет равным нулю. Такое состояние называют состоянием невесомости .

Состояние, при котором вес тела равен нулю, называют невесомостью.

Наконец, если ускорение лифта, направленное к Земле, станет больше ускорения свободного падения, тело окажется прижатым к потолку лифта. В этом случае вес тела изменит свое направление. Состояние невесомости исчезнет. В этом можно легко убедиться, если резко дернуть вниз банку с находящимся в ней предметом, закрыв верх банки ладонью, как показано на рис. 108.

Итоги

Весом тела называют силу, с которой это тело действует на поднес или опору, находясь относительно подвеса или опоры в неподвижном состоянии.

Вес тела в лифте, движущемся с направленным вверх относительно Земли ускорением, по модулю больше модуля силы тяжести. Такое явление называют перегрузкой .

Коэффициент перегрузки (перегрузка) – отношение веса тела, при перегрузке к силе тяжести, действующей на это тело.

Если вес тела равен нулю, то такое состояние называют невесомостью .

Вопросы

  1. Какую силу называют силой реакции опоры? Что называют весом тела?
  2. К чему приложен вес тела?
  3. Приведите примеры, когда вес тела: а) равен силе тяжести; б) равен нулю; в) больше силы тяжести; г) меньше силы тяжести.
  4. Что называют перегрузкой?
  5. Какое состояние называют невесомостью?

Упражнения

  1. Семиклассник Сергей стоит на напольных весах в комнате. Стрелка прибора установилась напротив деления 50 кг. Определите модуль веса Сергея. Ответьте на остальные три вопроса об этой силе.
  2. Найдите перегрузку, испытываемую космонавтом, который находится в ракете, поднимающейся вертикально вверх с ускорением a = Зg.
  3. С какой силой действует космонавт массой m = 100 кг на ракету, указанную в упражнении 2? Как называется эта сила?
  4. Найдите вес космонавта массой m = 100 кг в ракете, которая: а) стоит неподвижно на пусковой установке; б) поднимается с ускорением a = 4g, направленным вертикально вверх.
  5. Определите модули сил, действующих на гирю массой m = 2 кг, которая висит неподвижно На легкой нити, прикрепленной к потолку комнаты. Чему равны модули силы упругости, действующей со стороны нити: а) на гирю; б) на потолок? Чему равен вес гири? Указание: для ответа на поставленные вопросы воспользуйтесь законами Ньютона.
  6. Найдите вес груза массой m = 5 кг, подвешенного на нити к потолку скоростного лифта, если: а) лифт равномерно поднимается; б) лифт равномерно опускается; в) поднимающийся вверх со скоростью v = 2 м/с лифт начал торможение с ускорением a = 2 м/с 2 ; г) опускающийся вниз со скоростью v = 2 м/с лифт начал торможение с ускорением a = 2 м/с 2 ; д) лифт начал движение вверх с ускорением a = 2 м/с 2 ; е) лифт начал движение вниз с ускорением a = 2 м/с 2 .

Как определить силу реакции опоры

Статика – один из разделов современной физики, который изучает условия нахождения тел и систем в механическом равновесии. Для решения задач на равновесие важно знать, что такое сила реакции опоры. Данная статья посвящена подробному рассмотрению этого вопроса.

Второй и третий законы Ньютона

Прежде чем рассматривать определение силы реакции опоры, следует вспомнить о том, что вызывает движение тел.

Причиной нарушения механического равновесия является действие на тела внешних или внутренних сил. В результате этого действия тело приобретает определенное ускорение, которое вычисляется с помощью следующего равенства:

Эта запись известна как второй закон Ньютона. Здесь сила F является результирующей всех действующих на тело сил.

Если одно тело воздействует с некоторой силой F1¯ на второе тело, то второе оказывает действие на первое с точно такой же по абсолютной величине силой F2¯, но она направлена в противоположном направлении, чем F1¯. То есть справедливо равенство:

Эта запись является математическим выражением для третьего ньютоновского закона.

При решении задач с использованием этого закона школьники часто допускают ошибку, сравнивая эти силы. Например, лошадь везет телегу, при этом лошадь на телегу и телега на лошадь оказывают одинаковые по модулю силы. Почему же тогда вся система движется? Ответ на этот вопрос можно правильно дать, если вспомнить, что обе названные силы приложены к разным телам, поэтому они друг друга не уравновешивают.

Сила реакции опоры

Сначала дадим физическое определение этой силы, а затем поясним на примере, как она действует. Итак, силой нормальной реакции опоры называется сила, которая действует на тело со стороны поверхности. Например, мы поставили стакан с водой на стол. Чтобы стакан не двигался с ускорением свободного падения вниз, стол воздействует на него с силой, которая уравновешивает силу тяжести. Это и есть реакция опоры. Ее обычно обозначают буквой N.

Сила N – это контактная величина. Если имеется контакт между телами, то она появляется всегда. В примере выше значение величины N равно по модулю весу тела. Тем не менее это равенство является лишь частным случаем. Реакция опоры и вес тела – это совершенно разные силы, имеющие различную природу. Равенство между ними нарушается всегда, когда изменяется угол наклона плоскости, появляются дополнительные действующие силы, или когда система движется ускоренно.

Сила N называется нормальной потому, что она всегда направлена перпендикулярно плоскости поверхности.

Если говорить о третьем законе Ньютона, то в примере выше со стаканом воды на столе вес тела и нормальная сила N не являются действием и противодействием, поскольку обе они приложены к одному телу (стакану с водой).

Физическая причина появления силы N

Как было выяснено выше, сила реакции опоры препятствует проникновению одних твердых тел в другие. Почему появляется эта сила? Причина заключается в деформации. Любые твердые тела под воздействием нагрузки деформируются сначала упруго. Сила упругости стремится восстановить прежнюю форму тела, поэтому она оказывает выталкивающее воздействие, что проявляется в виде реакции опоры.

Если рассматривать вопрос на атомном уровне, то появление величины N – это результат действия принципа Паули. При небольшом сближении атомов их электронные оболочки начинают перекрываться, что приводит к появлению силы отталкивания.

Многим может показаться странным, что стакан с водой способен деформировать стол, но это так. Деформация настолько мала, что невооруженным глазом ее невозможно наблюдать.

Как вычислять силу N?

Сразу следует сказать, что какой-то определенной формулы силы реакции опоры не существует. Тем не менее имеется методика, применяя которую, можно определить N для совершенно любой системы взаимодействующих тел.

Методика определения величины N заключается в следующем:

  • сначала записывают второй закон Ньютона для данной системы, учитывая все действующие в ней силы;
  • находят результирующую проекцию всех сил на направление действия реакции опоры;
  • решение полученного уравнения Ньютона на отмеченное направление приведет к искомому значению N.

При составлении динамического уравнения следует внимательно и правильно расставлять знаки действующих сил.

Найти реакцию опоры можно также, если пользоваться не понятием сил, а понятием их моментов. Привлечение моментов сил справедливо и является удобным для систем, которые имеют точки или оси вращения.

Далее приведем два примера решения задач, в которых покажем, как пользоваться вторым ньютоновским законом и понятием момента силы для нахождения величины N.

Задача со стаканом на столе

Выше уже был приведен этот пример. Предположим, что пластиковый стакан объемом 250 мл наполнен водой. Его поставили на стол, а сверху на стакан положили книгу массой 300 грамм. Чему равна сила реакции опоры стола?

Запишем динамическое уравнение. Имеем:

Здесь P1 и P2 – вес стакана с водой и книги соответственно. Поскольку система находится в равновесии, то a=0. Учитывая, что вес тела равен силе тяжести, а также пренебрегая массой пластикового стакана, получаем:

Учитывая, что плотность воды равна 1 г/см 3 , и 1 мл равен 1 см 3 , получаем согласно выведенной формуле, что сила N равна 5,4 ньютона.

Задача с доской, двумя опорами и грузом

Доска, массой которой можно пренебречь, лежит на двух твердых опорах. Длина доски равна 2 метра. Чему будет равна сила реакции каждой опоры, если на эту доску посередине положить груз массой 3 кг?

Прежде чем переходить к решению задачи, следует ввести понятие момента силы. В физике этой величине соответствует произведение силы на длину рычага (расстояние от точки приложения силы до оси вращения). Система, имеющая ось вращения, будет находиться в равновесии, если суммарный момент сил равен нулю.

Возвращаясь к нашей задаче, вычислим суммарный момент сил относительно одной из опор (правой). Обозначим длину доски буквой L. Тогда момент силы тяжести груза будет равен:

Здесь L/2 – рычаг действия силы тяжести. Знак минус появился потому, что момент M1 осуществляет вращение против часовой стрелки.

Момент силы реакции опоры будет равен:

Поскольку система находится в равновесии, то сумма моментов должна быть равной нулю. Получаем:

Заметим, что от длины доски сила N не зависит.

Учитывая симметричность расположения груза на доске относительно опор, сила реакции левой опоры также будет равна 14,7 Н.

Тестирование онлайн

Что надо знать о силе

Сила – векторная величина. Необходимо знать точку приложения и направление каждой силы. Важно уметь определить какие именно силы действуют на тело и в каком направлении. Сила обозначается как , измеряется в Ньютонах. Для того, чтобы различать силы, их обозначают следующим образом

Ниже представлены основные силы, действующие в природе. Придумывать не существующие силы при решении задач нельзя!

Сил в природе много. Здесь рассмотрены силы, которые рассматриваются в школьном курсе физики при изучении динамики. А также упомянуты другие силы, которые будут рассмотрены в других разделах.

Сила тяжести

На каждое тело, находящееся на планете, действует гравитация Земли. Сила, с которой Земля притягивает каждое тело, определяется по формуле

Точка приложения находится в центре тяжести тела. Сила тяжести всегда направлена вертикально вниз.

Сила трения

Познакомимся с силой трения. Эта сила возникает при движении тел и соприкосновении двух поверхностей. Возникает сила в результате того, что поверхности, если рассмотреть под микроскопом, не являются гладкими, как кажутся. Определяется сила трения по формуле:

Сила приложена в точке соприкосновения двух поверхностей. Направлена в сторону противоположную движению.

Так как тело представляем в виде материальной точки, силу можно изображать с центра

Сила реакции опоры

Представим очень тяжелый предмет, лежащий на столе. Стол прогибается под тяжестью предмета. Но согласно третьему закону Ньютона стол воздействует на предмет с точно такой же силой, что и предмет на стол. Сила направлена противоположно силе, с которой предмет давит на стол. То есть вверх. Эта сила называется реакцией опоры. Название силы «говорит» реагирует опора. Эта сила возникает всегда, когда есть воздействие на опору. Природа ее возникновения на молекулярном уровне. Предмет как бы деформировал привычное положение и связи молекул (внутри стола), они, в свою очередь, стремятся вернуться в свое первоначальное состояние, «сопротивляются».

Абсолютно любое тело, даже очень легкое (например,карандаш, лежащий на столе), на микроуровне деформирует опору. Поэтому возникает реакция опоры.

Специальной формулы для нахождения этой силы нет. Обозначают ее буквой , но эта сила просто отдельный вид силы упругости, поэтому она может быть обозначена и как

Сила приложена в точке соприкосновения предмета с опорой. Направлена перпендикулярно опоре.

Так как тело представляем в виде материальной точки, силу можно изображать с центра

Сила упругости

Это сила возникает в результате деформации (изменения первоначального состояния вещества). Например, когда растягиваем пружину, мы увеличиваем расстояние между молекулами материала пружины. Когда сжимаем пружину – уменьшаем. Когда перекручиваем или сдвигаем. Во всех этих примерах возникает сила, которая препятствует деформации – сила упругости.

Сила упругости направлена противоположно деформации.

Так как тело представляем в виде материальной точки, силу можно изображать с центра

При последовательном соединении, например, пружин жесткость рассчитывается по формуле

При параллельном соединении жесткость

Жесткость образца. Модуль Юнга.

Модуль Юнга характеризует упругие свойства вещества. Это постоянная величина, зависящая только от материала, его физического состояния. Характеризует способность материала сопротивляться деформации растяжения или сжатия. Значение модуля Юнга табличное.

Подробнее о свойствах твердых тел здесь.

Вес тела

Вес тела – это сила, с которой предмет воздействует на опору. Вы скажете, так это же сила тяжести! Путаница происходит в следующем: действительно часто вес тела равен силе тяжести, но это силы совершенно разные. Сила тяжести – сила, которая возникает в результате взаимодействия с Землей. Вес – результат взаимодействия с опорой. Сила тяжести приложена в центре тяжести предмета, вес же – сила, которая приложена на опору (не на предмет)!

Формулы определения веса нет. Обозначается эта силы буквой .

Сила реакции опоры или сила упругости возникает в ответ на воздействие предмета на подвес или опору, поэтому вес тела всегда численно одинаков силе упругости, но имеет противоположное направление.

Сила реакции опоры и вес – силы одной природы, согласно 3 закону Ньютона они равны и противоположно направлены. Вес – это сила, которая действует на опору, а не на тело. Сила тяжести действует на тело.

Вес тела может быть не равен силе тяжести. Может быть как больше, так и меньше, а может быть и такое, что вес равен нулю. Это состояние называется невесомостью. Невесомость – состояние, когда предмет не взаимодействует с опорой, например, состояние полета: сила тяжести есть, а вес равен нулю!

Определить направление ускорения возможно, если определить, куда направлена равнодействующая сила

Обратите внимание, вес – сила, измеряется в Ньютонах. Как верно ответить на вопрос: «Сколько ты весишь»? Мы отвечаем 50 кг, называя не вес, а свою массу! В этом примере, наш вес равен силе тяжести, то есть примерно 500Н!

Перегрузка – отношение веса к силе тяжести

Сила Архимеда

Сила возникает в результате взаимодействия тела с жидкость (газом), при его погружении в жидкость (или газ). Эта сила выталкивает тело из воды (газа). Поэтому направлена вертикально вверх (выталкивает). Определяется по формуле:

В воздухе силой Архимеда пренебрегаем.

Если сила Архимеда равна силе тяжести, тело плавает. Если сила Архимеда больше, то оно поднимается на поверхность жидкости, если меньше – тонет.

Электрические силы

Существуют силы электрического происхождения. Возникают при наличии электрического заряда. Эти силы, такие как сила Кулона, сила Ампера, сила Лоренца, подробно рассмотрены в разделе Электричество.

Схематичное обозначение действующих на тело сил

Часто тело моделируют материальной точкой. Поэтому на схемах различные точки приложения переносят в одну точку – в центр, а тело изображают схематично кругом или прямоугольником.

Для того, чтобы верно обозначить силы, необходимо перечислить все тела, с которыми исследуемое тело взаимодействует. Определить, что происходит в результате взаимодействия с каждым: трение, деформация, притяжение или может быть отталкивание. Определить вид силы, верно обозначить направление. Внимание! Количество сил будет совпадать с числом тел, с которыми происходит взаимодействие.

Главное запомнить

1) Силы и их природа;
2) Направление сил;
3) Уметь обозначить действующие силы

Силы трения*

Различают внешнее (сухое) и внутреннее (вязкое) трение. Внешнее трение возникает между соприкасающимися твердыми поверхностями, внутреннее – между слоями жидкости или газа при их относительном движении. Существует три вида внешнего трения: трение покоя, трение скольжения и трение качения.

Трение качения определяется по формуле

Сила сопротивления возникает при движении тела в жидкости или в газе. Величина силы сопротивления зависит от размеров и формы тела, скорости его движения и свойств жидкости или газа. При небольших скоростях движения сила сопротивления пропорциональна скорости тела

При больших скоростях пропорциональна квадрату скорости

Взаимосвязь силы тяжести, закона гравитации и ускорения свободного падения*

Рассмотрим взаимное притяжение предмета и Земли. Между ними, согласно закону гравитации возникает сила

А сейчас сравним закон гравитации и силу тяжести

Величина ускорения свободного падения зависит от массы Земли и ее радиуса! Таким образом, можно высчитать, с каким ускорением будут падать предметы на Луне или на любой другой планете, используя массу и радиус той планеты.

Расстояние от центра Земли до полюсов меньше, чем до экватора. Поэтому и ускорение свободного падения на экваторе немного меньше, чем на полюсах. Вместе с тем, следует отметить, что основной причиной зависимости ускорения свободного падения от широты местности, является факт вращения Земли вокруг своей оси.

При удалении от поверхности Земли сила земного тяготения и ускорения свободного падения изменяются обратно пропорционально квадрату расстояния до центра Земли.

Расчет реакций относится к разделу физики с названием «Статика», которая рассматривает структуру и системы, находящиеся в покое.

Силой реакции опоры называется усилие противодействия опоры действующему на нее объекту, при этом она равна по модулю и противоположна по направлению усилию, с которым объект действует на опору, согласно третьему закону Ньютона.

Система между некоторой структурой и опорой, которая препятствует линейному или угловому перемещению этой структуры, называется системой опоры. Существует несколько типов опор:

  • Шарнир (валик) — опора первого порядка, ограничивающая смещение в пространстве в одном измерении и обладающее реакцией опоры перпендикулярной основанию.
  • Плоская опора — опора второго порядка, которая ограничивает перемещение в пространстве в двух измерениях (горизонтальном и вертикальном) и разрешает только движение вращения структуры.

Расчет равновесных систем связан с вычислением результирующего динамического момента. В ньютоновской (классической) механике момент силы определяется как векторное произведение усилия, действующего на опору, на вектор, образованный между точкой опоры и точкой приложения этого усилия. Момент силы также называют динамическим моментом или просто моментом.

Далее в статье приводится пример расчета реакции для наиболее распространенной задачи: балки с двумя опорами.

Решение задачи о реакции опоры балки

Как было сказано выше, балка с двумя опорами является типичной и наиболее простой задачей статики. Задача состоит в расчете реакций в точках А и В ввиду действующих на балку усилий.

Знание этих величин необходимо для правильного понимания диаграмм моментов и диаграмм сил данной системы, и является важной частью статики в школьных и университетских курсах. Существует компьютерная программа SkyCiv, которая предоставляет мощный инструмент по расчету таких реакций для различных равновесных систем.

Возвращаясь к поставленной выше задаче, напомним, что основным ее условием является статическое состояние, то есть отсутствие каких-либо линейных перемещений и вращений объектов. В простой физике последний факт означает, что сумма векторов всех усилий равна нулю (то есть сумма усилий, направленных вверх, равна таковым, направленным вниз). Вторым условием равновесия системы является равенство нулю динамических моментов, приложенных относительно определенной точки опоры.

Чтобы определить реакции подпорок балки, следуйте нижеизложенным двум способам решения задачи:

  • используя равенство нулю суммы динамических моментов;
  • используя равенство нулю суммы действующих усилий.

Первый способ: через моменты

Для начала нужно положить, что сумма всех моментов относительно точки реакции равна нулю, то есть ΣMi = 0, где Мi – момент усилия. Расчет таких моментов для нашей задачи очень прост, и состоит в перемножении действующих усилий на расстояния от точки их приложения до точки реакции.

Будем считать, что наша балка имеет длину 4 метра и расположена на двух подпорках А и В. Посредине балки вертикально вниз действует усилие в 20 кН, и нужно рассчитать реакции каждой подпорки, то есть Ay и By . Описанная задача представлена на рисунке.

Например, рассчитаем сумму всех динамических моментов относительно точки реакции В, учитывая ее равенство нулю в равновесии. Выбор точки В, относительно которой будет проводиться расчет, является произвольным, точно так же можно выбрать точку А. Таким образом, просуммируем все динамические моменты относительно точки В, полагая эту сумму равной нулю:

ΣMв = 0 = 20*2 – A y * 4 ==> A y = 10 кН.

Отметим, что в формуле выше мы выбрали положительное направление для моментов, действующих против часовой стрелки, и отрицательное направление для моментов, действующих по часовой стрелке. Такой выбор знаков моментом является наиболее общим, однако, вы можете выбрать и наоборот. Необходимо помнить, что всегда нужно использовать одно и то же соглашение на знак моментов, начиная сначала и следуя ему на протяжении всего решения конкретной задачи.

Таким образом, мы получили нашу первую формулу, из которой определили силу реакции опоры в точке А. Аналогичная формула запишется для определения реакции в точке В. В нашем случае, ввиду симметричности действующего вертикально вниз усилия в 20 кН относительно точек подпорок, реакция в точке В будет равна таковой в точке А, то есть 10 кН.

Второй способ: через силы

Для существования равновесия сумма всех вертикальных сил должна быть равна нулю, то есть ΣF y = 0, где индекс Y определяет конкретную вертикальную силу в системе. Помните, что в данном случае мы должны включать в расчет все действующие в системе силы. Принимая во внимание последний факт, проводим суммирование всех вертикальных сил, в итоге получаем следующую формулу:

ΣF y = 0 = A y + В y – 20 кН, откуда 0 = 10 кН + В y – 20 кН, и В y = 10 кН.

Так же, как и в случае моментов сил, силы являются векторными величинами и имеют знак, здесь мы приняли за положительные силы те, которые действуют вверх, и за отрицательные те, которые действуют вниз. Выбор знака остается за вами, однако, напоминаем, что этот выбор не должен изменяться в процессе решения задачи. Отметим, что в формуле выше мы использовали результат, полученный в предыдущем пункте, когда вычислили силу реакции Ay.

Таким образом, мы решили, поставленную в начале этого параграфа задачу о расчете сил реакций опоры балки, используя при этом две системы уравнений, уравнения момента силы и уравнения силы, и получили ответы: силы реакции в точках А и В равны между собой и составляют 10 кН. Напоминаем, что физический смысл полученного равенства заключается в том, что действующая на балку внешняя сила приложена точно посередине балки. В случае ее приложения в другой точке, приведенные формулы также будут действительны и процесс расчета остается тем же самым.

Видео

Эта видеоподборка поможет вам лучше разобраться в теме и закрепить полученные знания.

Сила реакции опоры формула в лифте. Определение опорных реакций. Отрывок, характеризующий Сила нормальной реакции

Сила реакции опоры относится к силам упругости, и всегда направлена перпендикулярно поверхности. Она противостоит любой силе, которая заставляет тело двигаться перпендикулярно опоре. Для того чтобы рассчитать ее нужно выявить и узнать числовое значение всех сил, которые действуют на тело, стоящее на опоре.

Вам понадобится

  • — весы;
  • — спидометр или радар;
  • — угломер.

Инструкция

  • Определите массу тела с помощью весов или любым другим способом. Если тело находится на горизонтальной поверхности (причем неважно, движется оно или пребывает в состоянии покоя), то сила реакции опоры равна силе тяжести действующей на тело. Для того чтобы рассчитать ее умножьте массу тела на ускорение свободного падения, которое равно 9,81 м/с² N=m g.
  • Когда тело движется по наклонной плоскости, направленной под углом к горизонту, сила реакции опоры находится под углом в силе тяжести. При этом она компенсирует только ту составляющую силы тяжести, которая действует перпендикулярно наклонной плоскости. Для расчета силы реакции опоры, с помощью угломера измерьте угол, под которым плоскость располагается к горизонту. Рассчитайте силу реакции опоры, перемножив массу тела на ускорение свободного падения и косинус угла, под которым плоскость находится к горизонту N=m g Cos(α).
  • В том случае, если тело движется по поверхности, которая представляет собой часть окружности с радиусом R, например, мост, пригорок то сила реакции опоры учитывает силу, действующую по направлению из центра окружности, с ускорением, равным центростремительному, действующую на тело. Чтобы рассчитать силу реакции опоры в верхней точке, от ускорения свободного падения отнимите отношение квадрата скорости к радиусу кривизны траектории.
  • Получившееся число умножьте на массу движущегося тела N=m (g-v²/R). Скорость должна быть измерена в метрах в секунду, а радиус в метрах. При определенной скорости значение ускорения, направленного от центра окружности, может сравняться, и даже превысить ускорение свободного падения, в этот момент сцепление тела с поверхностью пропадет, поэтому, например, автомобилистам, нужно четко контролировать скорость на таких участках дороги.
  • Если же кривизна направлена вниз, и траектория тела вогнутая, то рассчитайте силу реакции опоры, прибавив к ускорению свободного падения отношение квадрата скорости и радиуса кривизны траектории, а получившийся результат умножьте на массу тела N=m (g+v²/R).
  • Если известна сила трения и коэффициент трения, силу реакции опоры рассчитайте, поделив силу трения на этот коэффициент N=Fтр/μ.

Равномерное движение

S = v * t

S – путь, расстояние [м] (метр)

v – скорость [м/с] (метр в секунду)

t – время [ c ] (секунда)

Формула перевода скорости:

х км/ч= font-family:Arial»> м/с

Средняя скорость

v сред = EN-US»>s в – весь путь

t в – всё время

Плотность вещества

ρ= EN-US»>ρ – плотность

m – масса [кг] (килограмм)

V – объем [м3] (метр кубический)

Сила тяжести, вес и сила реакции опоры

Сила тяжести – сила притяжения к Земле. Приложена к телу. Направлена к центру Земли.

Вес – сила, с которой тело давит на опору или растягивает подвес. Приложена к телу. Направлена перпендикулярно опоре и параллельно подвесу вниз.

Сила реакции опоры – сила, с которой опора или подвес сопротивляется давлению или растяжению. Приложена к опоре или подвесу. Направлена перпендикулярно опоре или параллельно подвесу вверх.

F т =m*g; P=m*g*cosα; N=m*g*cosα

F т – сила тяжести [Н] (Ньютон)

P – вес [ Н ]

N – сила реакции опоры [Н]

m – масса [кг] (килограмм)

α – угол между плоскостью горизонта и плоскостью опоры [º,рад] (градус, радиан)

g≈9,8 м / с2

Сила упругости (Закон Гука)

F упр = k * x

F упр — сила упругости [Н] (Ньютон)

k – коэффициент жёсткости [Н/м] (Ньютон на метр)

x – удлинение/сжатие пружины [м] (метр)

Механическая работа

A=F*l*cosα

A – работа [Дж] (Джоуль)

F – сила [Н] (Ньютон)

l – расстояние, на котором действует сила [м] (метр)

α – угол между направлением силы и направлением движения [º,рад] (градус, радиан)

Частные случаи:

1)α=0, т. е. направление действия силы совпадает с направлением движения

A=F*l;

2) α = π /2=90 º, т. е. направление силы перпендикулярно направлению движения

A=0;

3) α = π =180 º, т. е. направление силы противоположно направлению движения

A =- F * l ;

Мощность

N = EN-US»>N – мощность [Вт] (Ватт)

A – работа [Дж] (Джоуль)

t – время [с] (секунда)

Давление в жидкостях и твёрдых телах

P = font-family:Arial»>; P = ρ * g * h

P – давление [Па] (Паскаль)

F – сила давления [Н] (Ньютон)

s – площадь основания [м2] (квадратный метр)

ρ – плотность материала/жидкости [кг/м3] (килограмм на метр кубический)

g – ускорение свободного падения [м/с2] (метр на секунду в квадрате)

h – высота предмета/столба жидкости [м] (метр)

Сила Архимеда

Сила Архимеда – сила, с которой жидкость или газ стремятся вытолкнуть погруженное в них тело.

F Арх = ρ ж * V погр * g

F Арх – сила Архимеда [Н] (Ньютон)

ρ ж – плотность жидкости/газа [кг/м3] (килограмм на метр кубический)

V погр – объем погруженной части тела [м3] (метр кубический)

g – ускорение свободного падения [м/с2] (метр на секунду в квадрате)

Условие плавания тел:

ρ ж ≥ρ т

ρ т – плотность материала тела [кг/м3] (килограмм на метр кубический)

Правило рычага

F 1 * l 1 = F 2 * l 2 (равновесие рычага)

F 1,2 – сила, действующая на рычаг [Н] (Ньютон)

l 1,2 – длина плеча рычага соответствующей силы [м] (метр)

Правило моментов

M = F * l

M – момент силы [Н*м] (Ньютон-метр)

F – сила [Н] (Ньютон)

l – длина (рычага) [м] (метр)

M1=M2 (равновесие)

Сила трения

F тр =µ* N

F тр – сила трения [Н] (Ньютон)

µ — коэффициент трения [ , %]

N – сила реакции опоры [Н] (Ньютон)

Энергия тела

E кин = font-family:Arial»>; E п = m * g * h

E кин – кинетическая энергия [Дж] (Джоуль)

m – масса тела [кг] (килограмм)

v – скорость тела [м/с] (метр в секунду)

Еп – потенциальная энергия [Дж] (Джоуль)

g – ускорение свободного падения [м/с2] (метр на секунду в квадрате)

h – высота над землей [м] (метр)

Закон сохранения энергии: Энергия не исчезает в никуда и не появляется из ниоткуда, она лишь переходит из одних форм в другие.

Статика — один из разделов современной физики, который изучает условия нахождения тел и систем в механическом равновесии. Для решения задач на равновесие важно знать, что такое сила реакции опоры. Данная статья посвящена подробному рассмотрению этого вопроса.

Второй и третий законы Ньютона

Прежде чем рассматривать определение силы реакции опоры, следует вспомнить о том, что вызывает движение тел.

Причиной нарушения механического равновесия является действие на тела внешних или внутренних сил. В результате этого действия тело приобретает определенное ускорение, которое вычисляется с помощью следующего равенства:

Эта запись известна как второй закон Ньютона. Здесь сила F является результирующей всех действующих на тело сил.

Если одно тело воздействует с некоторой силой F 1 ¯ на второе тело, то второе оказывает действие на первое с точно такой же по абсолютной величине силой F 2 ¯, но она направлена в противоположном направлении, чем F 1 ¯. То есть справедливо равенство:

Эта запись является математическим выражением для третьего ньютоновского закона.

При решении задач с использованием этого закона школьники часто допускают ошибку, сравнивая эти силы. Например, лошадь везет телегу, при этом лошадь на телегу и телега на лошадь оказывают одинаковые по модулю силы. Почему же тогда вся система движется? Ответ на этот вопрос можно правильно дать, если вспомнить, что обе названные силы приложены к разным телам, поэтому они друг друга не уравновешивают.

Сила реакции опоры

Сначала дадим физическое определение этой силы, а затем поясним на примере, как она действует. Итак, силой нормальной называется сила, которая действует на тело со стороны поверхности. Например, мы поставили стакан с водой на стол. Чтобы стакан не двигался с ускорением свободного падения вниз, стол воздействует на него с силой, которая уравновешивает силу тяжести. Это и есть реакция опоры. Ее обычно обозначают буквой N.

Сила N — это контактная величина. Если имеется контакт между телами, то она появляется всегда. В примере выше значение величины N равно по модулю весу тела. Тем не менее это равенство является лишь частным случаем. Реакция опоры и вес тела — это совершенно разные силы, имеющие различную природу. Равенство между ними нарушается всегда, когда изменяется угол наклона плоскости, появляются дополнительные действующие силы, или когда система движется ускоренно.

Сила N называется нормальной потому, что она всегда направлена перпендикулярно плоскости поверхности.

Если говорить о третьем законе Ньютона, то в примере выше со стаканом воды на столе вес тела и нормальная сила N не являются действием и противодействием, поскольку обе они приложены к одному телу (стакану с водой).

Физическая причина появления силы N

Как было выяснено выше, сила реакции опоры препятствует проникновению одних твердых тел в другие. Почему появляется эта сила? Причина заключается в деформации. Любые твердые тела под воздействием нагрузки деформируются сначала упруго. Сила упругости стремится восстановить прежнюю форму тела, поэтому она оказывает выталкивающее воздействие, что проявляется в виде реакции опоры.

Если рассматривать вопрос на атомном уровне, то появление величины N — это результат действия принципа Паули. При небольшом сближении атомов их электронные оболочки начинают перекрываться, что приводит к появлению силы отталкивания.

Многим может показаться странным, что стакан с водой способен деформировать стол, но это так. Деформация настолько мала, что невооруженным глазом ее невозможно наблюдать.

Как вычислять силу N?

Сразу следует сказать, что какой-то определенной формулы силы реакции опоры не существует. Тем не менее имеется методика, применяя которую, можно определить N для совершенно любой системы взаимодействующих тел.

Методика определения величины N заключается в следующем:

  • сначала записывают второй закон Ньютона для данной системы, учитывая все действующие в ней силы;
  • находят результирующую проекцию всех сил на направление действия реакции опоры;
  • решение полученного уравнения Ньютона на отмеченное направление приведет к искомому значению N.

При составлении динамического уравнения следует внимательно и правильно расставлять знаки действующих сил.

Найти реакцию опоры можно также, если пользоваться не понятием сил, а понятием их моментов. Привлечение моментов сил справедливо и является удобным для систем, которые имеют точки или оси вращения.

Задача со стаканом на столе

Выше уже был приведен этот пример. Предположим, что пластиковый стакан объемом 250 мл наполнен водой. Его поставили на стол, а сверху на стакан положили книгу массой 300 грамм. Чему равна сила реакции опоры стола?

Запишем динамическое уравнение. Имеем:

Здесь P 1 и P 2 — вес стакана с водой и книги соответственно. Поскольку система находится в равновесии, то a=0. Учитывая, что вес тела равен силе тяжести, а также пренебрегая массой пластикового стакана, получаем:

m 1 *g + m 2 *g — N = 0 =>

N = (m 1 + m 2)*g

Учитывая, что плотность воды равна 1 г/см 3 , и 1 мл равен 1 см 3 , получаем согласно выведенной формуле, что сила N равна 5,4 ньютона.

Задача с доской, двумя опорами и грузом

Доска, массой которой можно пренебречь, лежит на двух твердых опорах. Длина доски равна 2 метра. Чему будет равна сила реакции каждой опоры, если на эту доску посередине положить груз массой 3 кг?

Прежде чем переходить к решению задачи, следует ввести понятие момента силы. В физике этой величине соответствует произведение силы на длину рычага (расстояние от точки приложения силы до оси вращения). Система, имеющая ось вращения, будет находиться в равновесии, если суммарный момент сил равен нулю.

Возвращаясь к нашей задаче, вычислим суммарный относительно одной из опор (правой). Обозначим длину доски буквой L. Тогда момент силы тяжести груза будет равен:

Здесь L/2 — рычаг действия силы тяжести. Знак минус появился потому, что момент M 1 осуществляет вращение против часовой стрелки.

Момент силы реакции опоры будет равен:

Поскольку система находится в равновесии, то сумма моментов должна быть равной нулю. Получаем:

M 1 + M 2 = 0 =>

N*L + (-m*g*L/2) = 0 =>

N = m*g/2 = 3*9,81/2 = 14,7 Н

Заметим, что от длины доски сила N не зависит.

Учитывая симметричность расположения груза на доске относительно опор, сила реакции левой опоры также будет равна 14,7 Н.

Силу действующую на тело со стороны опоры (или подвеса), называют силой реакции опоры. При соприкосновении тел сила реакции опоры направлена перпендикулярно поверхности соприкосновения. Если тело лежит на горизонтальном неподвижном столе, сила реакции опоры направлена вертикально вверх и уравновешивает силу тяжести:

Wikimedia Foundation . 2010 .

Смотреть что такое «Сила нормальной реакции опоры» в других словарях:

    Сила трения скольжения силы, возникающие между соприкасающимися телами при их относительном движении. Если между телами отсутствует жидкая или газообразная прослойка (смазка), то такое трение называется сухим. В противном случае, трение… … Википедия

    Запрос «сила» перенаправляется сюда; см. также другие значения. Сила Размерность LMT−2 Единицы измерения СИ … Википедия

    Запрос «сила» перенаправляется сюда; см. также другие значения. Сила Размерность LMT−2 Единицы измерения СИ ньютон … Википедия

    Закон Амонтона Кулона эмпирический закон, устанавливающий связь между поверхностной силой трения, возникающей при относительном скольжении тела, с силой нормальной реакции, действующей на тело со стороны поверхности. Сила трения,… … Википедия

    Силы трения скольжения силы, возникающие между соприкасающимися телами при их относительном движении. Если между телами отсутствует жидкая или газообразная прослойка (смазка), то такое трение называется сухим. В противном случае, трение… … Википедия

    Трение покоя, трение сцепления сила, возникающая между двумя контактирующими телами и препятствующая возникновению относительного движения. Эту силу необходимо преодолеть для того, чтобы привести два контактирующих тела в движение друг… … Википедия

    Сюда перенаправляется запрос «Прямохождение». На эту тему нужна отдельная статья. Ходьба человека наиболее естественная локомоция человека. Автоматизированный двигательный акт, осуществляющийся в результате сложной координированной деятельности… … Википедия

    Цикл ходьбы: опора на одну ногу двуопорный период опора на другую ногу… Ходьба человека наиболее естественная локомоция человека. Автоматизированный двигательный акт, осуществляющийся в результате сложной координированной деятельности скелетных … Википедия

    Сила трения при скольжении тела о поверхность не зависит от площади соприкосновения тела с поверхностью, но зависит от силы нормальной реакции этого тела и от состояния окружающей среды. Сила трения скольжения возникает при скольжении данного… … Википедия

    Закон Амонтона Кулона сила трения при скольжении тела о поверхность не зависит от площади соприкосновения тела с поверхностью, но зависит от силы нормальной реакции этого тела и от состояния окружающей среды. Сила трения скольжения возникает при… … Википедия

Тестирование онлайн

Что надо знать о силе

Сила — векторная величина. Необходимо знать точку приложения и направление каждой силы. Важно уметь определить какие именно силы действуют на тело и в каком направлении. Сила обозначается как , измеряется в Ньютонах. Для того, чтобы различать силы, их обозначают следующим образом

Ниже представлены основные силы, действующие в природе. Придумывать не существующие силы при решении задач нельзя!

Сил в природе много. Здесь рассмотрены силы, которые рассматриваются в школьном курсе физики при изучении динамики. А также упомянуты другие силы, которые будут рассмотрены в других разделах.

Сила тяжести

На каждое тело, находящееся на планете, действует гравитация Земли. Сила, с которой Земля притягивает каждое тело, определяется по формуле

Точка приложения находится в центре тяжести тела. Сила тяжести всегда направлена вертикально вниз .

Сила трения

Познакомимся с силой трения. Эта сила возникает при движении тел и соприкосновении двух поверхностей. Возникает сила в результате того, что поверхности, если рассмотреть под микроскопом, не являются гладкими, как кажутся. Определяется сила трения по формуле:

Сила приложена в точке соприкосновения двух поверхностей. Направлена в сторону противоположную движению.

Сила реакции опоры

Представим очень тяжелый предмет, лежащий на столе. Стол прогибается под тяжестью предмета. Но согласно третьему закону Ньютона стол воздействует на предмет с точно такой же силой, что и предмет на стол. Сила направлена противоположно силе, с которой предмет давит на стол. То есть вверх. Эта сила называется реакцией опоры. Название силы «говорит» реагирует опора . Эта сила возникает всегда, когда есть воздействие на опору. Природа ее возникновения на молекулярном уровне. Предмет как бы деформировал привычное положение и связи молекул (внутри стола), они, в свою очередь, стремятся вернуться в свое первоначальное состояние, «сопротивляются».

Абсолютно любое тело, даже очень легкое (например,карандаш, лежащий на столе), на микроуровне деформирует опору. Поэтому возникает реакция опоры.

Специальной формулы для нахождения этой силы нет. Обозначают ее буквой , но эта сила просто отдельный вид силы упругости, поэтому она может быть обозначена и как

Сила приложена в точке соприкосновения предмета с опорой. Направлена перпендикулярно опоре.

Так как тело представляем в виде материальной точки, силу можно изображать с центра

Сила упругости

Это сила возникает в результате деформации (изменения первоначального состояния вещества). Например, когда растягиваем пружину, мы увеличиваем расстояние между молекулами материала пружины. Когда сжимаем пружину — уменьшаем. Когда перекручиваем или сдвигаем. Во всех этих примерах возникает сила, которая препятствует деформации — сила упругости.


Сила упругости направлена противоположно деформации.

При последовательном соединении, например, пружин жесткость рассчитывается по формуле

При параллельном соединении жесткость

Жесткость образца. Модуль Юнга.

Модуль Юнга характеризует упругие свойства вещества. Это постоянная величина, зависящая только от материала, его физического состояния. Характеризует способность материала сопротивляться деформации растяжения или сжатия. Значение модуля Юнга табличное.

Подробнее о свойствах твердых тел здесь.

Вес тела — это сила, с которой предмет воздействует на опору. Вы скажете, так это же сила тяжести! Путаница происходит в следующем: действительно часто вес тела равен силе тяжести, но это силы совершенно разные. Сила тяжести — сила, которая возникает в результате взаимодействия с Землей. Вес — результат взаимодействия с опорой. Сила тяжести приложена в центре тяжести предмета, вес же — сила, которая приложена на опору (не на предмет)!

Формулы определения веса нет. Обозначается эта силы буквой .

Сила реакции опоры или сила упругости возникает в ответ на воздействие предмета на подвес или опору, поэтому вес тела всегда численно одинаков силе упругости, но имеет противоположное направление.

Сила реакции опоры и вес — силы одной природы, согласно 3 закону Ньютона они равны и противоположно направлены. Вес — это сила, которая действует на опору, а не на тело. Сила тяжести действует на тело.

Вес тела может быть не равен силе тяжести. Может быть как больше, так и меньше, а может быть и такое, что вес равен нулю. Это состояние называется невесомостью . Невесомость — состояние, когда предмет не взаимодействует с опорой, например, состояние полета: сила тяжести есть, а вес равен нулю!

Определить направление ускорения возможно, если определить, куда направлена равнодействующая сила

Обратите внимание, вес — сила, измеряется в Ньютонах. Как верно ответить на вопрос: «Сколько ты весишь»? Мы отвечаем 50 кг, называя не вес, а свою массу! В этом примере, наш вес равен силе тяжести, то есть примерно 500Н!

Перегрузка — отношение веса к силе тяжести

Сила Архимеда

Сила возникает в результате взаимодействия тела с жидкость (газом), при его погружении в жидкость (или газ). Эта сила выталкивает тело из воды (газа). Поэтому направлена вертикально вверх (выталкивает). Определяется по формуле:

В воздухе силой Архимеда пренебрегаем.

Если сила Архимеда равна силе тяжести, тело плавает. Если сила Архимеда больше, то оно поднимается на поверхность жидкости, если меньше — тонет.

Электрические силы

Существуют силы электрического происхождения. Возникают при наличии электрического заряда. Эти силы, такие как сила Кулона, сила Ампера, сила Лоренца, подробно рассмотрены в разделе Электричество.

Схематичное обозначение действующих на тело сил

Часто тело моделируют материальной точкой. Поэтому на схемах различные точки приложения переносят в одну точку — в центр, а тело изображают схематично кругом или прямоугольником.

Для того, чтобы верно обозначить силы, необходимо перечислить все тела, с которыми исследуемое тело взаимодействует. Определить, что происходит в результате взаимодействия с каждым: трение, деформация, притяжение или может быть отталкивание. Определить вид силы, верно обозначить направление. Внимание! Количество сил будет совпадать с числом тел, с которыми происходит взаимодействие.

Главное запомнить

1) Силы и их природа;
2) Направление сил;
3) Уметь обозначить действующие силы

Силы трения*

Различают внешнее (сухое) и внутреннее (вязкое) трение. Внешнее трение возникает между соприкасающимися твердыми поверхностями, внутреннее — между слоями жидкости или газа при их относительном движении. Существует три вида внешнего трения: трение покоя, трение скольжения и трение качения.

Трение качения определяется по формуле

Сила сопротивления возникает при движении тела в жидкости или в газе. Величина силы сопротивления зависит от размеров и формы тела, скорости его движения и свойств жидкости или газа. При небольших скоростях движения сила сопротивления пропорциональна скорости тела

При больших скоростях пропорциональна квадрату скорости

Взаимосвязь силы тяжести, закона гравитации и ускорения свободного падения*

Рассмотрим взаимное притяжение предмета и Земли. Между ними, согласно закону гравитации возникает сила

А сейчас сравним закон гравитации и силу тяжести

Величина ускорения свободного падения зависит от массы Земли и ее радиуса! Таким образом, можно высчитать, с каким ускорением будут падать предметы на Луне или на любой другой планете, используя массу и радиус той планеты.

Расстояние от центра Земли до полюсов меньше, чем до экватора. Поэтому и ускорение свободного падения на экваторе немного меньше, чем на полюсах. Вместе с тем, следует отметить, что основной причиной зависимости ускорения свободного падения от широты местности, является факт вращения Земли вокруг своей оси.

При удалении от поверхности Земли сила земного тяготения и ускорения свободного падения изменяются обратно пропорционально квадрату расстояния до центра Земли.

Сила реакции опоры. Вес

Положим камень на горизонтальную крышку стола, стоящего на Земле (рис. 104). Поскольку ускорение камня относительно Земли равно пулю, то по второму закону Ньютона сумма действующих на него сил равна нулю. Следовательно, действие на камень силы тяжести m · g должно компенсироваться какими-то другими силами. Ясно, что под действием камня крышка стола деформируется. Поэтому со стороны стола на камень действует сила упругости. Если считать, что камень взаимодействует лишь с Землей и крышкой стола, то сила упругости должна уравновешивать силу тяжести: F упр = -m · g. Эту силу упругости называют силой реакции опоры и обозначают латинской буквой N. Так как ускорение свободного падения направлено вертикально вниз, сила N направлена вертикально вверх – перпендикулярно поверхности крышки стола.

Поскольку крышка стола действует на камень, то по третьему закону Ньютона и камень действует на крышку стола силой P = -N (рис. 105). Эту силу называют весом .

Весом тела называют силу, с которой это тело действует на подвес или опору, находясь относительно подвеса или опоры в неподвижном состоянии.

Ясно, что в рассмотренном случае вес камня равен силе тяжести: P = m · g. Это будет верно для любого тела, покоящегося на подвесе (опоре) относительно Земли (рис. 106). Очевидно, что в этом случае точка крепления подвеса (или опора) неподвижна относительно Земли.

Для тела, покоящегося на неподвижном относительно Земли подвесе (опоре), вес тела равен силе тяжести.

Вес тела также будет равен действующей на тело силе тяжести в случае, если тело и подвес (опора) движутся относительно Земли равномерно прямолинейно.

Если же тело и подвес (опора) движутся относительно Земли с ускорением так, что тело остается неподвижным относительно подвеса (опоры), то вес тела не будет равен силе тяжести.

Рассмотрим пример. Пусть тело массой m лежит на полу лифта, ускорение a которого направлено вертикально вверх (рис. 107). Будем считать, что на тело действуют только сила тяжести m · g и сила реакции пола N. (Вес тела действует не на тело, а на опору – пол лифта.) В системе отсчета, неподвижной относительно Земли, тело на полу лифта движется вместе с лифтом с ускорением a. В соответствии со вторым законом Ньютона произведение массы тела на ускорение равно сумме всех действующих на тело сил. Поэтому: m · a = N — m · g.

Следовательно, N = m · a + m · g = m · (g + a). Значит, если лифт имеет ускорение, направленное вертикально вверх, то модуль силы N реакции пола будет больше модуля силы тяжести. В самом деле, сила реакции пола должна не только скомпенсировать действие силы тяжести, но и придать телу ускорение в положительном направлении оси X.

Сила N – это сила, с которой пол лифта действует на тело. По третьему закону Ньютона тело действует на пол с силой P, модуль которой равен модулю N, но направлена сила P в противоположную сторону. Эта сила является весом тела в движущемся лифте. Модуль этой силы P = N = m · (g + a). Таким образом, в лифте, движущемся с направленным вверх относительно Земли ускорением, модуль веса тела больше модуля силы тяжести .

Такое явление называют перегрузкой .

Например, пусть ускорение а лифта направлено вертикально вверх и его значение равно g, т. е. a = g. В этом случае модуль веса тела – силы, действующей на пол лифта, – будет равен P = m · (g + a) = m · (g + g) = 2m · g. То есть вес тела при этом будет в два раза больше, чем в лифте, который относительно Земли покоится или движется равномерно прямолинейно.

Для тела на подвесе (или опоре), движущемся с ускорением относительно Земли, направленным вертикально вверх, вес тела больше силы тяжести.

Отношение веса тела в движущемся ускоренно относительно Земли лифте к весу этого же тела в покоящемся или движущемся равномерно прямолинейно лифте называют коэффициентом перегрузки или, более кратко, перегрузкой .

Коэффициент перегрузки (перегрузка) – отношение веса тела при перегрузке к силе тяжести, действующей на тело.

В рассмотренном выше случае перегрузка равна 2. Понятно, что если бы ускорение лифта было направлено вверх и его значение было равно a = 2g, то коэффициент перегрузки был бы равен 3.

Теперь представим себе, что тело массой m лежит на полу лифта, ускорение которого a относительно Земли направлено вертикально вниз (противоположно оси X). Если модуль a ускорения лифта будет меньше модуля ускорения свободного падения, то сила реакции пола лифта по-прежнему будет направлена вверх, в положительном направлении оси X, а ее модуль будет равен N = m · (g — a). Следовательно, модуль веса тела будет равен P = N = m · (g — a), т. е. будет меньше модуля силы тяжести. Таким образом, тело будет давить на пол лифта с силой, модуль которой меньше модуля силы тяжести.

Это ощущение знакомо каждому, кто ездил на скоростном лифте или качался на больших качелях. При движении вниз из верхней точки вы чувствуете, что ваше давление на опору уменьшается. Если же ускорение опоры положительно (лифт и качели начинают подниматься), вас сильнее прижимает к опоре.

Если ускорение лифта относительно Земли будет направлено вниз и равно по модулю ускорению свободного падения (лифт свободно падает), то сила реакции пола станет равной нулю: N = m · (g — a) = m · (g — g) = 0. В этом случае пол лифта перестанет давить на лежащее на нем тело. Следовательно, согласно третьему закону Ньютона и тело не будет давить на пол лифта, совершая вместе с лифтом свободное падение. Вес тела станет равным нулю. Такое состояние называют состоянием невесомости .

Состояние, при котором вес тела равен нулю, называют невесомостью.

Наконец, если ускорение лифта, направленное к Земле, станет больше ускорения свободного падения, тело окажется прижатым к потолку лифта. В этом случае вес тела изменит свое направление. Состояние невесомости исчезнет. В этом можно легко убедиться, если резко дернуть вниз банку с находящимся в ней предметом, закрыв верх банки ладонью, как показано на рис. 108.

Итоги

Весом тела называют силу, с которой это тело действует на поднес или опору, находясь относительно подвеса или опоры в неподвижном состоянии.

Вес тела в лифте, движущемся с направленным вверх относительно Земли ускорением, по модулю больше модуля силы тяжести. Такое явление называют перегрузкой .

Коэффициент перегрузки (перегрузка) – отношение веса тела, при перегрузке к силе тяжести, действующей на это тело.

Если вес тела равен нулю, то такое состояние называют невесомостью .

Вопросы

  1. Какую силу называют силой реакции опоры? Что называют весом тела?
  2. К чему приложен вес тела?
  3. Приведите примеры, когда вес тела: а) равен силе тяжести; б) равен нулю; в) больше силы тяжести; г) меньше силы тяжести.
  4. Что называют перегрузкой?
  5. Какое состояние называют невесомостью?
  6. Упражнения

  7. Семиклассник Сергей стоит на напольных весах в комнате. Стрелка прибора установилась напротив деления 50 кг. Определите модуль веса Сергея. Ответьте на остальные три вопроса об этой силе.
  8. Найдите перегрузку, испытываемую космонавтом, который находится в ракете, поднимающейся вертикально вверх с ускорением a = Зg.
  9. С какой силой действует космонавт массой m = 100 кг на ракету, указанную в упражнении 2? Как называется эта сила?
  10. Найдите вес космонавта массой m = 100 кг в ракете, которая: а) стоит неподвижно на пусковой установке; б) поднимается с ускорением a = 4g, направленным вертикально вверх.
  11. Определите модули сил, действующих на гирю массой m = 2 кг, которая висит неподвижно На легкой нити, прикрепленной к потолку комнаты. Чему равны модули силы упругости, действующей со стороны нити: а) на гирю; б) на потолок? Чему равен вес гири? Указание: для ответа на поставленные вопросы воспользуйтесь законами Ньютона.
  12. Найдите вес груза массой m = 5 кг, подвешенного на нити к потолку скоростного лифта, если: а) лифт равномерно поднимается; б) лифт равномерно опускается; в) поднимающийся вверх со скоростью v = 2 м/с лифт начал торможение с ускорением a = 2 м/с 2 ; г) опускающийся вниз со скоростью v = 2 м/с лифт начал торможение с ускорением a = 2 м/с 2 ; д) лифт начал движение вверх с ускорением a = 2 м/с 2 ; е) лифт начал движение вниз с ускорением a = 2 м/с 2 .

ЗАКОНЫ НЬЮТОНА ВИДЫ СИЛ. Виды сил Сила упругости Сила трения Сила тяжести Сила Архимеда Сила натяжения нити Сила реакции опоры Вес тела Сила всемирного. — презентация

Презентация на тему: » ЗАКОНЫ НЬЮТОНА ВИДЫ СИЛ. Виды сил Сила упругости Сила трения Сила тяжести Сила Архимеда Сила натяжения нити Сила реакции опоры Вес тела Сила всемирного.» — Транскрипт:

1 ЗАКОНЫ НЬЮТОНА ВИДЫ СИЛ

2 Виды сил Сила упругости Сила трения Сила тяжести Сила Архимеда Сила натяжения нити Сила реакции опоры Вес тела Сила всемирного тяготения

3 Законы Ньютона. 1 ЗаконЗакон2 ЗаконЗакон3 Закон

4 1 закон Ньютона. Существуют системы отсчёта, называемые инерциальными, относительно которых свободные тела движутся равномерно и прямолинейно. Законы

5 2 закон Ньютона. Произведение массы тела на его ускорение равно сумме действующих на тело сил. Законы

6 3 закон Ньютона. Силы, с которыми тела действуют друг на друга, равны по модулям и направлены по одной прямой в противоположные стороны Законы

7 СССС ииии лллл аааа в в в в сссс ееее мммм ииии рррр нннн оооо гггг оооо тттт яяяя гггг оооо тттт ееее нннн ииии яяяя. G – гравитационная постоянная. m – масса тела r – расстояние между центрами тел.

8 СССС ииии лллл аааа в в в в сссс ееее мммм ииии рррр нннн оооо гггг оооо т т т т яяяя гггг оооо тттт ееее нннн ииии яяяя – – – – пппп рррр ииии тттт яяяя жжжж ееее нннн ииии ееее т т т т ееее лллл д д д д рррр уууу гггг к к к к д д д д рррр уууу гггг уууу. НННН аааа пппп рррр аааа вввв лллл ееее нннн аааа п п п п оооо п п п п рррр яяяя мммм оооо йййй. сссс оооо ееее дддд ииии нннн яяяя юююю щщщщ ееее йййй ц ц ц ц ееее нннн тттт рррр ыыыы т т т т ееее лллл.

9 СССС ииии лллл аааа н н н н аааа тттт яяяя жжжж ееее нннн ииии яяяя н н н н ииии тттт ииии T-действие подвеса на тело направлено вдоль нити

10 N NN Сила реакции опоры – (N) – действие опоры на тело, направлено перпендикулярно опоры. Сила реакции опоры

11 Сила трения Сила трения Это действие поверхности на движущиеся или пытающиеся сдвинуться тело, направлено против движения или возможного движения. Если тело не двигается то сила трения равна приложенной силе. Если тело двигается или только начинает движение, то сила трения находится по формуле: — коэффициэнт трения N — сила реакции опоры Сила трения

12 Сила упругости Сила упругости Сила упругости- это действие упруго- деформированного тела. Направлена против деформации.

13 Действие тела на опору или подвес ВЕС |P|=|N| |P|=|T|

14 Сила Архимеда Сила Архимеда-это сила с которой жидкость действует на погруженное в неё тело. СИЛА АРХИМЕДА

15 СИЛА ТЯЖЕСТИ Сила тяжести- это сила с которой земля действует на тело, направлена к центру земли.

Сила реакции опоры закон

Рис. 7. Силы натяжения

Если реакция опоры становится равной нулю, говорят, что тело находится в состоянии невесомости . В состоянии невесомости тело движется только под действием силы тяжести.

1.2.3. Инертность и инерция. Инерциальные системы отсчета.

Первый закон Ньютона

Опыт показывает, что любое тело противится попыткам изменить его состояние вне зависимости от того, движется оно или покоится. Это свойство тел называется инертностью . Понятие инертности нельзя путать с инерцией тел. Инерция тел проявляется в том, что в отсутствие внешних воздействий тела находятся в состоянии покоя или прямолинейного и равномерного движения до тех пор, пока какое – либо внешнее воздействие не изменит этого состояния. Инерция, в отличие от инертности, не имеет количественной характеристики.

Задачи динамики решаются с помощью трех основных законов, получивших название законов Ньютона. Законы Ньютона выполняются в инерциальных системах отсчета. Инерциальные системы отсчета (ИСО) — это системы отсчета, в которых тела, не подверженные воздействию других тел, движутся без ускорения, то есть прямолинейно и равномерно, или покоятся.

Первый закон Ньютона (закон инерции): существуют такие системы отсчета (так называемые, инерциальные системы), для которых любая материальная точка в отсутствие внешних воздействий движется равномерно и прямолинейно или находится в состоянии покоя. Согласно принципу относительности Галилея все механические явления в различных инерциальных системах отсчета протекают одинаково и никакими механическими опытами невозможно установить, покоится данная система отсчета или движется прямолинейно и равномерно.

1.2.4. Второй закон Ньютона. Импульс тела и импульс силы.

Закон сохранения импульса. Третий закон Ньютона

Второй закон Ньютона: ускорение, приобретаемое материальной точкой под действием одной или нескольких сил, прямо пропорционально действующей силе (или равнодействующей всех сил), обратно пропорционально массе материальной точки и по направлению совпадает с направлением действующей силы (или равнодействующей):

. (8)

Второй закон Ньютона имеет еще одну форму записи. Введем понятие импульса тела.

Импульс тела (или просто, импульс) – мера механического движения, определяемая произведением массы тела
на его скорость , т.е.,
. Запишем второй закон Ньютона — основное уравнение динамики поступательного движения:

Заменим сумму сил на ее равнодействующую
и запись второго закона Ньютона принимает следующий вид:

, (9)

а сам второй закон Ньютона закон может быть сформулирован еще и так: скорость изменения импульса определяет действующую на тело силу .

Преобразуем последнюю формулу:
. Величина
получила названиеимпульса силы. Импульс силы
определяется изменением импульса тела
.

Механическая система тел, на которую не действуют внешние силы, называется замкнутой (или изолированной).

Закон сохранения импульса : импульс замкнутой системы тел есть величина постоянная.

Третий закон Ньютона: силы, возникающие при взаимодействии тел, равны по величине, противоположны по направлению и приложены к разным телам (рис. 8):

. (10)

Рис. 8. Третий закон Ньютона

Из 3-го закона Ньютона следует, что при взаимодействии тел силы возникают парами. В полную систему законов динамики кроме законов Ньютона необходимо включить принцип независимости действия сил: действие какой-либо силы не зависит от присутствия или отсутствия других сил; совместное действие нескольких сил равно сумме независимых действий отдельных сил.

Сила нормальной реакции опоры

Силу действующую на тело со стороны опоры (или подвеса), называют силой реакции опоры. При соприкосновении тел сила реакции опоры направлена перпендикулярно поверхности соприкосновения. Если тело лежит на горизонтальном неподвижном столе, сила реакции опоры направлена вертикально вверх и уравновешивает силу тяжести:

Wikimedia Foundation . 2010 .

Смотреть что такое «Сила нормальной реакции опоры» в других словарях:

Сила трения скольжения — Сила трения скольжения силы, возникающие между соприкасающимися телами при их относительном движении. Если между телами отсутствует жидкая или газообразная прослойка (смазка), то такое трение называется сухим. В противном случае, трение… … Википедия

Сила (физическая величина) — Запрос «сила» перенаправляется сюда; см. также другие значения. Сила Размерность LMT−2 Единицы измерения СИ … Википедия

Сила — Запрос «сила» перенаправляется сюда; см. также другие значения. Сила Размерность LMT−2 Единицы измерения СИ ньютон … Википедия

Закон Амонтона — Закон Амонтона Кулона эмпирический закон, устанавливающий связь между поверхностной силой трения, возникающей при относительном скольжении тела, с силой нормальной реакции, действующей на тело со стороны поверхности. Сила трения,… … Википедия

Закон трения — Силы трения скольжения силы, возникающие между соприкасающимися телами при их относительном движении. Если между телами отсутствует жидкая или газообразная прослойка (смазка), то такое трение называется сухим. В противном случае, трение… … Википедия

Трение покоя — Трение покоя, трение сцепления сила, возникающая между двумя контактирующими телами и препятствующая возникновению относительного движения. Эту силу необходимо преодолеть для того, чтобы привести два контактирующих тела в движение друг… … Википедия

Ходьба человека — Сюда перенаправляется запрос «Прямохождение». На эту тему нужна отдельная статья. Ходьба человека наиболее естественная локомоция человека. Автоматизированный двигательный акт, осуществляющийся в результате сложной координированной деятельности… … Википедия

Прямохождение — Цикл ходьбы: опора на одну ногу двуопорный период опора на другую ногу. Ходьба человека наиболее естественная локомоция человека. Автоматизированный двигательный акт, осуществляющийся в результате сложной координированной деятельности скелетных … Википедия

Закон Амонтона — Кулона — сила трения при скольжении тела о поверхность не зависит от площади соприкосновения тела с поверхностью, но зависит от силы нормальной реакции этого тела и от состояния окружающей среды. Сила трения скольжения возникает при скольжении данного… … Википедия

Закон Кулона (механика) — Закон Амонтона Кулона сила трения при скольжении тела о поверхность не зависит от площади соприкосновения тела с поверхностью, но зависит от силы нормальной реакции этого тела и от состояния окружающей среды. Сила трения скольжения возникает при… … Википедия

Сила тяжести, трения, реакции опоры, упругости, Архимеда, сопротивления, вес. Направление, точка приложения, природа возникновения

Тестирование онлайн

Что надо знать о силе

Сила — векторная величина. Необходимо знать точку приложения и направление каждой силы. Важно уметь определить какие именно силы действуют на тело и в каком направлении. Сила обозначается как , измеряется в Ньютонах. Для того, чтобы различать силы, их обозначают следующим образом

Ниже представлены основные силы, действующие в природе. Придумывать не существующие силы при решении задач нельзя!

Сил в природе много. Здесь рассмотрены силы, которые рассматриваются в школьном курсе физики при изучении динамики. А также упомянуты другие силы, которые будут рассмотрены в других разделах.

Сила тяжести

На каждое тело, находящееся на планете, действует гравитация Земли. Сила, с которой Земля притягивает каждое тело, определяется по формуле

Точка приложения находится в центре тяжести тела. Сила тяжести всегда направлена вертикально вниз.

Сила трения

Познакомимся с силой трения. Эта сила возникает при движении тел и соприкосновении двух поверхностей. Возникает сила в результате того, что поверхности, если рассмотреть под микроскопом, не являются гладкими, как кажутся. Определяется сила трения по формуле:

Сила приложена в точке соприкосновения двух поверхностей. Направлена в сторону противоположную движению.

Так как тело представляем в виде материальной точки, силу можно изображать с центра

Сила реакции опоры

Представим очень тяжелый предмет, лежащий на столе. Стол прогибается под тяжестью предмета. Но согласно третьему закону Ньютона стол воздействует на предмет с точно такой же силой, что и предмет на стол. Сила направлена противоположно силе, с которой предмет давит на стол. То есть вверх. Эта сила называется реакцией опоры. Название силы «говорит» реагирует опора. Эта сила возникает всегда, когда есть воздействие на опору. Природа ее возникновения на молекулярном уровне. Предмет как бы деформировал привычное положение и связи молекул (внутри стола), они, в свою очередь, стремятся вернуться в свое первоначальное состояние, «сопротивляются».

Абсолютно любое тело, даже очень легкое (например,карандаш, лежащий на столе), на микроуровне деформирует опору. Поэтому возникает реакция опоры.

Специальной формулы для нахождения этой силы нет. Обозначают ее буквой , но эта сила просто отдельный вид силы упругости, поэтому она может быть обозначена и как

Сила приложена в точке соприкосновения предмета с опорой. Направлена перпендикулярно опоре.

Так как тело представляем в виде материальной точки, силу можно изображать с центра

Сила упругости

Это сила возникает в результате деформации (изменения первоначального состояния вещества). Например, когда растягиваем пружину, мы увеличиваем расстояние между молекулами материала пружины. Когда сжимаем пружину — уменьшаем. Когда перекручиваем или сдвигаем. Во всех этих примерах возникает сила, которая препятствует деформации — сила упругости.

Закон Гука

Сила упругости направлена противоположно деформации.

Так как тело представляем в виде материальной точки, силу можно изображать с центра

При последовательном соединении, например, пружин жесткость рассчитывается по формуле

При параллельном соединении жесткость

Жесткость образца. Модуль Юнга.

Модуль Юнга характеризует упругие свойства вещества. Это постоянная величина, зависящая только от материала, его физического состояния. Характеризует способность материала сопротивляться деформации растяжения или сжатия. Значение модуля Юнга табличное.

Подробнее о свойствах твердых тел здесь.

Вес тела

Вес тела — это сила, с которой предмет воздействует на опору. Вы скажете, так это же сила тяжести! Путаница происходит в следующем: действительно часто вес тела равен силе тяжести, но это силы совершенно разные. Сила тяжести — сила, которая возникает в результате взаимодействия с Землей. Вес — результат взаимодействия с опорой. Сила тяжести приложена в центре тяжести предмета, вес же — сила, которая приложена на опору (не на предмет)!

Формулы определения веса нет. Обозначается эта силы буквой .

Сила реакции опоры или сила упругости возникает в ответ на воздействие предмета на подвес или опору, поэтому вес тела всегда численно одинаков силе упругости, но имеет противоположное направление.


Сила реакции опоры и вес — силы одной природы, согласно 3 закону Ньютона они равны и противоположно направлены. Вес — это сила, которая действует на опору, а не на тело. Сила тяжести действует на тело.

Вес тела может быть не равен силе тяжести. Может быть как больше, так и меньше, а может быть и такое, что вес равен нулю. Это состояние называется невесомостью. Невесомость — состояние, когда предмет не взаимодействует с опорой, например, состояние полета: сила тяжести есть, а вес равен нулю!


Определить направление ускорения возможно, если определить, куда направлена равнодействующая сила

Обратите внимание, вес — сила, измеряется в Ньютонах. Как верно ответить на вопрос: «Сколько ты весишь»? Мы отвечаем 50 кг, называя не вес, а свою массу! В этом примере, наш вес равен силе тяжести, то есть примерно 500Н!

Перегрузка — отношение веса к силе тяжести

Сила Архимеда

Сила возникает в результате взаимодействия тела с жидкость (газом), при его погружении в жидкость (или газ). Эта сила выталкивает тело из воды (газа). Поэтому направлена вертикально вверх (выталкивает). Определяется по формуле:

В воздухе силой Архимеда пренебрегаем.

Если сила Архимеда равна силе тяжести, тело плавает. Если сила Архимеда больше, то оно поднимается на поверхность жидкости, если меньше — тонет.


Электрические силы

Существуют силы электрического происхождения. Возникают при наличии электрического заряда. Эти силы, такие как сила Кулона, сила Ампера, сила Лоренца, подробно рассмотрены в разделе Электричество.

Схематичное обозначение действующих на тело сил

Часто тело моделируют материальной точкой. Поэтому на схемах различные точки приложения переносят в одну точку — в центр, а тело изображают схематично кругом или прямоугольником.

Для того, чтобы верно обозначить силы, необходимо перечислить все тела, с которыми исследуемое тело взаимодействует. Определить, что происходит в результате взаимодействия с каждым: трение, деформация, притяжение или может быть отталкивание. Определить вид силы, верно обозначить направление. Внимание! Количество сил будет совпадать с числом тел, с которыми происходит взаимодействие.

Главное запомнить

1) Силы и их природа;
2) Направление сил;
3) Уметь обозначить действующие силы

Рассмотрим взаимное притяжение предмета и Земли. Между ними, согласно закону гравитации возникает сила

А сейчас сравним закон гравитации и силу тяжести

Величина ускорения свободного падения зависит от массы Земли и ее радиуса! Таким образом, можно высчитать, с каким ускорением будут падать предметы на Луне или на любой другой планете, используя массу и радиус той планеты.

Расстояние от центра Земли до полюсов меньше, чем до экватора. Поэтому и ускорение свободного падения на экваторе немного меньше, чем на полюсах. Вместе с тем, следует отметить, что основной причиной зависимости ускорения свободного падения от широты местности, является факт вращения Земли вокруг своей оси.

При удалении от поверхности Земли сила земного тяготения и ускорения свободного падения изменяются обратно пропорционально квадрату расстояния до центра Земли.

Закон силы трения: объясняем сложную тему простыми словами

Определение силы трения

Когда мы говорим «абсолютно гладкая поверхность» — это значит, что между ней и телом нет трения. Такая ситуация в реальной жизни практически невозможна. Избавиться от трения полностью невероятно трудно.

Чаще при слове «трение» нам приходит в голову его «тёмная» сторона —  из-за трения скрипят и  прекращают качаться качели, изнашиваются детали машин. Но представьте, что вы стоите на идеально гладкой поверхности, и вам надо идти или бежать. Вот тут трение бы, несомненно, пригодилось. Без него вы не сможете сделать ни шагу, ведь между ботинком и поверхностью нет сцепления, и вам не от чего оттолкнуться, чтобы двигаться вперёд.

Трение — это взаимодействие, которое возникает в плоскости контакта поверхностей соприкасающихся тел.
Сила трения — это величина, которая характеризует это взаимодействие по величине и направлению. 

Основная особенность: сила трения приложена к обоим телам, поверхности которых соприкасаются, и направлена в сторону, противоположную мгновенной скорости движения тел друг относительно друга. Поэтому тела, свободно скользящие по какой-либо горизонтальной поверхности, в конце концов остановятся. Чтобы тело двигалось по горизонтальной поверхности без торможения, к нему надо прикладывать усилие, противоположное и хотя бы равное силе трения. В этом заключается суть силы трения. 

Откуда берётся трение

Трение возникает по двум причинам:

  1. Все тела имеют шероховатости. Даже у очень хорошо отшлифованных металлов в электронный микроскоп видны неровности. Абсолютно гладкие поверхности бывают только в идеальном мире задач, в которых трением можно пренебречь. Именно упругие и неупругие деформации неровностей при контакте трущихся поверхностей формируют силу трения. 
  2. Между атомами и молекулами поверхностей тел действуют электромагнитные силы притяжения и отталкивания. Таким образом, сила трения имеет электромагнитную природу.

Виды силы трения


В зависимости от вида трущихся поверхностей, различают сухое и вязкое трение. В свою очередь, оба подразделяются на другие виды силы трения.

  1. Сухое трение возникает в области контакта поверхностей твёрдых тел в отсутствие жидкой или газообразной прослойки. Этот вид трения может возникать даже в состоянии покоя или в результате перекатывания одного тела по другому, поэтому здесь выделяют три вида силы трения:
  • трение скольжения,
  • трение покоя,
  • трение качения.  
  1. Вязкое трение возникает при движении твёрдого тела в жидкости или газе. Оно препятствует движению лодки, которая скользит по реке, или воздействует на летящий самолёт со стороны воздуха. Интересная особенность вязкого трения в том, что отсутствует трение покоя. Попробуйте сдвинуть пальцем лежащий на земле деревянный брус и проделайте тот же эксперимент, опустив брус на воду. Чтобы сдвинуть брус с места в воде, будет достаточно сколь угодно малой силы. Однако по мере роста скорости силы вязкого трения сильно увеличиваются.

Сила трения покоя 


Рассмотрим силу трения покоя подробнее.

Обычная ситуация: на кухне имеется холодильник,  его нужно переставить на другое место.

Когда никто не пытается двигать холодильник, стоящий на горизонтальном полу, трения между ним и полом нет. Но как только его начинают толкать, коварная сила трения покоя тут же возникает и полностью компенсирует усилие. Причина её возникновения — те самые неровности соприкасающихся поверхностей, которые деформируясь, препятствуют движению холодильника. Поднатужились, увеличили силу,  приложенную к холодильнику, но он не поддался и остался на месте. Это означает, что сила трения покоя возрастает вместе с увеличением внешнего воздействия, оставаясь равной по модулю приложенной силе, ведь увеличиваются деформации неровностей.

Пока силы равны,  холодильник остаётся на месте:

Сила трения, которая действует между поверхностями покоящихся тел и препятствует возникновению движения, называется силой трения покоя

Сила трения скольжения


Что же делать с холодильником и можно ли победить силу трения покоя? Не будет же она расти до бесконечности? 

Зовём на помощь друга, и вдвоём уже удаётся передвинуть холодильник. Получается, чтобы тело двигалось, нужно приложить силу, большую, чем самая большая сила трения покоя: 

Теперь на движущийся холодильник действует сила трения скольжения. Она возникает при относительном движении контактирующих твёрдых тел.

Итак, сила трения покоя может меняться от нуля до некоторого максимального значения — Fтр. пок. макс  И если приложенная сила больше,  чем Fтр. пок. макс, то у холодильника появляется шанс сдвинуться с места.

Теперь, после начала движения, можно прекратить наращивать усилие и ещё  одного друга можно не звать. Чтобы холодильник продолжал двигаться равномерно, достаточно прикладывать силу, равную силе трения скольжения: 

Как рассчитать и измерить силу трения


Чтобы понять, как измеряется сила трения, нужно понять, какие факторы влияют на величину силы трения. Почему так трудно двигать холодильник?

Самое очевидное — его масса играет первостепенную роль. Можно вытащить из него все продукты и тем самым уменьшить его массу, и, следовательно, силу давления холодильника на опору (пол). Пустой холодильник сдвинуть с места гораздо легче!
Следовательно, чем меньше сила нормального давления тела на поверхность опоры, тем меньше и сила трения. Опора действует на тело с точно такой же силой, что и тело на опору, только направленной в противоположную сторону. 

Сила реакции опоры обозначается N. Можно сделать вывод

Второй фактор, влияющий на величину силы трения, — материал и степень обработки соприкасающихся поверхностей. Так, двигать холодильник по бетонному полу гораздо тяжелее, чем по ламинату. Зависимость силы трения от рода и качества обработки материала обеих соприкасающихся поверхностей выражают через коэффициент трения.  

Коэффициент трения обозначается буквой μ (греческая буква «мю»). Коэффициент определяется отношением силы трения к силе нормального давления. 

Он чаще всего попадает в интервал  от нуля до единицы, не имеет размерности и определяется экспериментально.

Можно предположить, что сила трения зависит также от площади соприкасающихся поверхностей. Однако, положив холодильник набок, мы не облегчим себе задачу.

Ещё Леонардо да Винчи экспериментально доказал, что сила трения не зависит от площади соприкасающихся поверхностей при прочих равных условиях.  

Сила трения скольжения, возникающая при контакте твёрдого тела с поверхностью другого твёрдого тела прямо пропорциональна силе нормального давления и не зависит от площади контакта. 

Этот факт отражён в законе Амонтона-Кулона, который можно записать формулой:

где  μ — коэффициент трения, N — сила нормальной реакции опоры.

Для тела, движущегося по горизонтальной поверхности, сила реакции опоры по модулю равна весу тела: 

Сила трения качения


Ещё древние строители заметили, что если тяжёлый предмет водрузить на колёсики, то сдвинуть с места и затем  катить его будет гораздо легче, чем тянуть волоком. Вот бы пригодилась эта древняя мудрость, когда мы тянули холодильник!  Однако всё равно нужно толкать или тянуть тело, чтобы оно не остановилось. Значит, на него действует сила трения качения. Это сила сопротивления движению при перекатывании одного тела по поверхности другого.

Причина трения качения — деформация катка и опорной поверхности. Сила трения качения может быть в сотни раз меньше силы трения скольжения при той же силе давления на поверхность. Примерами уменьшения силы трения за счёт подмены трения скольжения на трение качения служат такие приспособления, как подшипники, колёсики у чемоданов и сумок, ролики на прокатных станах.

Направление силы трения

Сила трения скольжения всегда направлена противоположно скорости относительного движения соприкасающихся тел. Важно помнить, что на каждое из соприкасающихся тел действует своя сила трения.

Бывают ситуации, когда сила трения не препятствует движению, а совсем наоборот.

Представьте, что на ленте транспортёра лежит чемодан. Лента трогается с места, и чемодан движется вместе с ней. Сила трения между лентой и чемоданом оказалась достаточной, чтобы преодолеть инерцию чемодана, и эти тела движутся как одно целое. На чемодан действует сила трения покоя, возникающая при взаимодействии соприкасающихся поверхностей, которая направлена по ходу движения ленты транспортёра.

 Если бы лента была абсолютно гладкой, то чемодан начал бы скользить по ней, стремясь сохранить своё состояние покоя. Напомним, что это явление называется инерцией.

Сила трения покоя, помогающая нам ходить и бегать, также направлена не против движения, а вперёд по ходу перемещения. При повороте же автомобиля  сила трения покоя и вовсе направлена к  центру окружности. 

Для того чтобы понять, как направлена сила трения покоя, нужно предположить, в каком направлении стало бы двигаться тело, будь поверхность идеально гладкой. Сила трения покоя в этом случае будет направлена как раз в противоположную сторону. Пример, лестница у стены.

Подведём итоги


  1. Сила трения покоя меняется от нуля до максимального значения 0 < Fтр.покоя < Fтр.пок.макс  в зависимости от внешнего воздействия.
  2. Максимальная сила трения покоя почти равна силе трения скольжения, лишь немного её превышая. Можно приближенно считать, что Fтр. = Fтр.пок.макс 
  3. Силу трения скольжения можно рассчитать по формуле Fтр. = μ ⋅ N,  где  μ — коэффициент трения, N — сила нормальной реакции опоры.
  4. При равномерном прямолинейном скольжении по горизонтальной поверхности сила тяги равна силе трения скольжения Fтр. = Fтяги.
  5. Коэффициент трения μ зависит от рода и степени обработки  поверхностей 0 < μ < 1 . 
  6. При одинаковых силе нормального давления и коэффициенте трения сила трения качения всегда меньше силы трения скольжения.
Учите физику вместе с домашней онлайн-школой «Фоксфорда»! По промокоду
PHYSICS72020 вы получите бесплатный доступ к курсу физики 7 класса, в котором изучается закон силы трения. 

Задачи на силу трения


Проверьте, насколько хорошо вы разобрались в теме «Сила трения», — решите несколько задач. Решение — приведено ниже. Но чур не смотреть, пока не попробуете разобраться сами.

  1. Однажды в день открытия железной дороги произошёл конфуз: угодливый чиновник, желая выслужиться перед Николаем I, приказал выкрасить рельсы белой масляной краской. Какая возникла проблема и как её удалось решить с помощью сажи?
  2. В один зимний день бабушка Нюра катала внука Алексея по заснеженной горизонтальной дороге. Чему равен коэффициент трения полозьев о снег, если сила трения, действующая на санки, равна 250 Н, а их масса вместе с Алексеем составляет 50 кг?
  3. На брусок массой m = 5 кг, находящийся на горизонтальной шероховатой поверхности μ = 0,7, начинает действовать сила F = 25 Н, направленная вдоль плоскости. Чему при этом равна сила трения, действующая на брусок?

Решения

  1. Масляная краска снизила коэффициент трения между колёсами и рельсами, что привело к пробуксовке, поезд не смог двигаться вперёд. Посыпав рельсы сажей, удалось решить проблему, так как коэффициент трения увеличился, и колёса перестали буксовать.
  2. Санки находятся в движении, следовательно, на них будет действовать сила трения скольжения, численно равная Fтр. = μ ⋅ N, где N — сила реакции опоры, которая, при условии горизонтальной поверхности, равняется весу санок с мальчиком: N = m ⋅ g.  Получаем формулу Fтр. = μ ⋅ m ⋅ g  , откуда выразим искомую величину 

Ответ задачи зависит от того, сдвинется ли брусок под действием внешнего воздействия. Поэтому вначале узнаем значение силы, которую нужно приложить к бруску для скольжения. Это будет максимально возможная сила трения покоя, определяющаяся по формуле Fтр. = μ ⋅ N , где N = mg (при условии горизонтальной поверхности). Подставляя значения, получаем, что Fтр. = 35 Н. Данное значение больше прикладываемой силы, следовательно брусок не сдвинется с места. Тогда сила трения покоя будет равна внешней силе: Fтр. = F = 25 H .

Как решать 2 задание ЕГЭ по физике, примеры решения (Ростов-на-Дону)

Из последних КИМов ЕГЭ по физике следует, что задание 2 относится к разделу «Динамика» и может содержать расчетные задачи по следующим темам: «Законы Ньютона, закон всемирного тяготения, закон Гука, сила трения».

Основные формулы, которые необходимо знать для успешного решения задания 2.

Сила тяжести

m — масса тела

g=10 м/с2ускорение свободного падения

Сила упругости

Δx – удлинение пружины

k – коэффициент жесткости пружины

Сила трения

µ — коэффициент трения

N – сила реакции опоры

Сила Архимеда (выталкивающая сила)

Vобъём погруженной части тела

g=10 м/с2ускорение свободного падения

Сила притяжения между телами (закон Всемирного тяготения)

G = 6,67*10-11 Н*м2/кг2 – гравитационная постоянная

m1 и m2 - массы взаимодействующих тел

r – расстояние между телами

Второй закон Ньютона

m – масса тела

R – равнодействующая всех сил, действующих на тело

a – ускорение, с которым движется тело под действием этих сил

При решении задач из раздела «Динамика» желательно придерживаться следующего алгоритма решения:

1. Сделать рисунок, на котором указать вектора всех сил, действующих на тело.

2. Если тело двигается с ускорением, указать направление этого ускорения. Если тело покоится или двигается равномерно, его ускорение a=0.

3. Составить уравнение движения (второй закон Ньютона) для рассматриваемого тела в его векторном виде.

3. Выбрать систему координат и спроецировать полученное уравнение на выбранные оси координат.

4. Расшифровать неизвестные величины, вошедшие в уравнение движения.

5. Решить полученную систему уравнений.

Задание 2 – это расчётные задачи базового уровня сложности, и для решения некоторых из них этот алгоритм будет чересчур подробным и перегруженным, так как их можно решить и без вспомогательного рисунка или даже без записи второго закона Ньютона. Это касается, например, заданий, в которых на тело действует только одна сила. Но привычка решать задания по приведенному выше алгоритму поможет ученикам успешно справиться с расчетными задачами по разделу «Динамика» повышенного и высокого уровней сложности – такие задания могут стоять в ЕГЭ под номерами 25 и 29.

Ответом на задание 2 является число, именно его нужно вписать в бланк ответов 1, не указывая единицы измерения.

Примеры решения

1. (ЕГЭ-2019)

Пружина жёсткостью 2*104 Н/м одним концом закреплена в штативе. На какую величину она растянется под действием силы 400 Н?

Ответ: ___________________________ см.

Решение:

Сделаем чертёж

Пружина под действием силы F привели в растянутое состояние. Кроме растягивающей силы F и силы упругости , стремящейся вернуть пружину в нерастянутое состояние, больше никакие силы на нее не действуют.

Запишем проекции сил на вертикальную ось Oy

F=Fупр

По закону Гука, сила упругости Fупр = kx, следовательно,

kкоэффициент жёсткости пружины, Δxеё удлинение.

Выразим величину растяжения пружины

Ответ: 2

  1. (ЕГЭ – 2020. Вариант 1 досрочного ЕГЭ)

Тело движется по горизонтальной плоскости. Нормальная составляющая силы воздействия тела на плоскость равна 40 Н, сила трения равна 10 Н. Определите коэффициент трения скольжения.

Ответ: _______ .

Решение:

Силу трения можно найти по формуле

Fтр= µN,

где N – сила реакции опоры, или по-другому нормальная составляющая силы воздействия тела на плоскость.

Ответ: 0,25.

  1. (ЕГЭ – 2020. Демонстрационный вариант)

Два одинаковых маленьких шарика массой m каждый, расстояние между центрами которых равно r, притягиваются друг к другу с силами, равными по модулю 0,2 пН. Каков модуль сил гравитационного притяжения двух других шариков, если масса каждого из них равна 2m, а расстояние между их центрами равно 2r?

Ответ: _______ пН.

Решение:

По закону Всемирного тяготения шары массами m1и m2, находящиеся друг от друга на расстоянии r, притягиваются друг к другу с силой

.

В первом случае

Во втором случае

Ответ: 0,2

  1. (ЕГЭ – 2019. Демонстрационный вариант)

По горизонтальному полу по прямой равномерно тянут ящик, приложив к нему горизонтальную силу 35 Н. Коэффициент трения скольжения между полом и ящиком равен 0,25. Чему равна масса ящика?

Ответ _______ кг.

Решение:

Сделаем чертёж, на котором обозначим все силы, действующие на тело.

По второму закону Ньютона, равнодействующая всех сил, действующих на тело, будет равна нулю, так как по условию задачи тело движется равномерно, то есть ускорение тела a=0.

Запишем это в проекциях на оси Ox и Oy

Ox: Fтр – F = 0,

Oy: N — m g=0.

Откуда N = mg, следовательно,

Fтр = µ N = µ mg.

Масса тела

Ответ: 14

  1. (ЕГЭ – 2018)

К пружине подвесили груз массой 150 г, вследствие чего пружина удлинилась на 1 см. Чему будет равно удлинение этой пружины, если к ней подвесить груз 450 г?

Ответ: __________ см.

Решение:

Переведём единицы измерения физических величин в систему СИ

m1 = 150 г = 0,15 кг, m2 = 450 г = 0,45 кг, Δx=1 см = 0,01 м.

Сделаем чертёж, на котором обозначим все силы, действующие на тело.

На тело действует сила тяжести (Fт = mg), направленная вертикально вниз, и сила упругости со стороны пружины (Fупр = k Δx), направленная вертикально вверх.

В проекции на вертикальную ось Oy.

Fт =Fупр

mg = kΔx (1)

kкоэффициент жёсткости пружины, Δxеё удлинение.

Найдём, чему равен коэффициент жёсткости пружины

Выразим из выражения (1) удлинение пружины во втором случае

Ответ: 3

РЕКОМЕНДУЕМЫЕ ТОВАРЫ

Сила реакции опоры. Вес | Физика

Положим камень на горизонтальную крышку стола, стоящего на Земле (рис. 104). Поскольку ускорение камня относительно Земли равно пулю, то по второму закону Ньютона сумма действующих на него сил равна нулю. Следовательно, действие на камень силы тяжести m · g должно компенсироваться какими-то другими силами. Ясно, что под действием камня крышка стола деформируется. Поэтому со стороны стола на камень действует сила упругости. Если считать, что камень взаимодействует лишь с Землей и крышкой стола, то сила упругости должна уравновешивать силу тяжести: Fупр = -m · g. Эту силу упругости называют силой реакции опоры и обозначают латинской буквой N. Так как ускорение свободного падения направлено вертикально вниз, сила N направлена вертикально вверх – перпендикулярно поверхности крышки стола.

Поскольку крышка стола действует на камень, то по третьему закону Ньютона и камень действует на крышку стола силой P = -N (рис. 105). Эту силу называют весом.

Весом тела называют силу, с которой это тело действует на подвес или опору, находясь относительно подвеса или опоры в неподвижном состоянии.

Ясно, что в рассмотренном случае вес камня равен силе тяжести: P = m · g. Это будет верно для любого тела, покоящегося на подвесе (опоре) относительно Земли (рис. 106). Очевидно, что в этом случае точка крепления подвеса (или опора) неподвижна относительно Земли.

Для тела, покоящегося на неподвижном относительно Земли подвесе (опоре), вес тела равен силе тяжести.

Вес тела также будет равен действующей на тело силе тяжести в случае, если тело и подвес (опора) движутся относительно Земли равномерно прямолинейно.

Если же тело и подвес (опора) движутся относительно Земли с ускорением так, что тело остается неподвижным относительно подвеса (опоры), то вес тела не будет равен силе тяжести.

Рассмотрим пример. Пусть тело массой m лежит на полу лифта, ускорение a которого направлено вертикально вверх (рис. 107). Будем считать, что на тело действуют только сила тяжести m · g и сила реакции пола N. (Вес тела действует не на тело, а на опору – пол лифта.) В системе отсчета, неподвижной относительно Земли, тело на полу лифта движется вместе с лифтом с ускорением a. В соответствии со вторым законом Ньютона произведение массы тела на ускорение равно сумме всех действующих на тело сил. Поэтому: m · a = N — m · g.

Следовательно, N = m · a + m · g = m · (g + a). Значит, если лифт имеет ускорение, направленное вертикально вверх, то модуль силы N реакции пола будет больше модуля силы тяжести. В самом деле, сила реакции пола должна не только скомпенсировать действие силы тяжести, но и придать телу ускорение в положительном направлении оси X.

Сила N – это сила, с которой пол лифта действует на тело. По третьему закону Ньютона тело действует на пол с силой P, модуль которой равен модулю N, но направлена сила P в противоположную сторону. Эта сила является весом тела в движущемся лифте. Модуль этой силы P = N = m · (g + a). Таким образом, в лифте, движущемся с направленным вверх относительно Земли ускорением, модуль веса тела больше модуля силы тяжести.

Такое явление называют перегрузкой.

Например, пусть ускорение а лифта направлено вертикально вверх и его значение равно g, т. е. a = g. В этом случае модуль веса тела – силы, действующей на пол лифта, – будет равен P = m · (g + a) = m · (g + g) = 2m · g. То есть вес тела при этом будет в два раза больше, чем в лифте, который относительно Земли покоится или движется равномерно прямолинейно.

Для тела на подвесе (или опоре), движущемся с ускорением относительно Земли, направленным вертикально вверх, вес тела больше силы тяжести.

Отношение веса тела в движущемся ускоренно относительно Земли лифте к весу этого же тела в покоящемся или движущемся равномерно прямолинейно лифте называют коэффициентом перегрузки или, более кратко, перегрузкой.

Коэффициент перегрузки (перегрузка) – отношение веса тела при перегрузке к силе тяжести, действующей на тело.

В рассмотренном выше случае перегрузка равна 2. Понятно, что если бы ускорение лифта было направлено вверх и его значение было равно a = 2g, то коэффициент перегрузки был бы равен 3.

Теперь представим себе, что тело массой m лежит на полу лифта, ускорение которого a относительно Земли направлено вертикально вниз (противоположно оси X). Если модуль a ускорения лифта будет меньше модуля ускорения свободного падения, то сила реакции пола лифта по-прежнему будет направлена вверх, в положительном направлении оси X, а ее модуль будет равен N = m · (g — a). Следовательно, модуль веса тела будет равен P = N = m · (g — a), т. е. будет меньше модуля силы тяжести. Таким образом, тело будет давить на пол лифта с силой, модуль которой меньше модуля силы тяжести.

Это ощущение знакомо каждому, кто ездил на скоростном лифте или качался на больших качелях. При движении вниз из верхней точки вы чувствуете, что ваше давление на опору уменьшается. Если же ускорение опоры положительно (лифт и качели начинают подниматься), вас сильнее прижимает к опоре.

Если ускорение лифта относительно Земли будет направлено вниз и равно по модулю ускорению свободного падения (лифт свободно падает), то сила реакции пола станет равной нулю: N = m · (g — a) = m · (g — g) = 0. В этом случае пол лифта перестанет давить на лежащее на нем тело. Следовательно, согласно третьему закону Ньютона и тело не будет давить на пол лифта, совершая вместе с лифтом свободное падение. Вес тела станет равным нулю. Такое состояние называют состоянием невесомости.

Состояние, при котором вес тела равен нулю, называют невесомостью.

Наконец, если ускорение лифта, направленное к Земле, станет больше ускорения свободного падения, тело окажется прижатым к потолку лифта. В этом случае вес тела изменит свое направление. Состояние невесомости исчезнет. В этом можно легко убедиться, если резко дернуть вниз банку с находящимся в ней предметом, закрыв верх банки ладонью, как показано на рис. 108.

Итоги

Весом тела называют силу, с которой это тело действует на поднес или опору, находясь относительно подвеса или опоры в неподвижном состоянии.

Вес тела в лифте, движущемся с направленным вверх относительно Земли ускорением, по модулю больше модуля силы тяжести. Такое явление называют перегрузкой.

Коэффициент перегрузки (перегрузка) – отношение веса тела, при перегрузке к силе тяжести, действующей на это тело.

Если вес тела равен нулю, то такое состояние называют невесомостью.

Вопросы

  1. Какую силу называют силой реакции опоры? Что называют весом тела?
  2. К чему приложен вес тела?
  3. Приведите примеры, когда вес тела: а) равен силе тяжести; б) равен нулю; в) больше силы тяжести; г) меньше силы тяжести.
  4. Что называют перегрузкой?
  5. Какое состояние называют невесомостью?

Упражнения

  1. Семиклассник Сергей стоит на напольных весах в комнате. Стрелка прибора установилась напротив деления 50 кг. Определите модуль веса Сергея. Ответьте на остальные три вопроса об этой силе.
  2. Найдите перегрузку, испытываемую космонавтом, который находится в ракете, поднимающейся вертикально вверх с ускорением a = Зg.
  3. С какой силой действует космонавт массой m = 100 кг на ракету, указанную в упражнении 2? Как называется эта сила?
  4. Найдите вес космонавта массой m = 100 кг в ракете, которая: а) стоит неподвижно на пусковой установке; б) поднимается с ускорением a = 4g, направленным вертикально вверх.
  5. Определите модули сил, действующих на гирю массой m = 2 кг, которая висит неподвижно На легкой нити, прикрепленной к потолку комнаты. Чему равны модули силы упругости, действующей со стороны нити: а) на гирю; б) на потолок? Чему равен вес гири? Указание: для ответа на поставленные вопросы воспользуйтесь законами Ньютона.
  6. Найдите вес груза массой m = 5 кг, подвешенного на нити к потолку скоростного лифта, если: а) лифт равномерно поднимается; б) лифт равномерно опускается; в) поднимающийся вверх со скоростью v = 2 м/с лифт начал торможение с ускорением a = 2 м/с2; г) опускающийся вниз со скоростью v = 2 м/с лифт начал торможение с ускорением a = 2 м/с2; д) лифт начал движение вверх с ускорением a = 2 м/с2; е) лифт начал движение вниз с ускорением a = 2 м/с2.

Реакция опоры — обзор

Пример 4.4

Рассчитайте реакции опоры в балке с опорой, показанной на рис. 4.7.

Рисунок 4.7. Использование принципа виртуальной работы для расчета опорных реакций

К балке прикладывается только вертикальная нагрузка, поэтому возникают только вертикальные реакции, R A и R C .

Предположим, что лучу в точке C задано небольшое воображаемое, то есть виртуальное смещение, Δ v , C , в направлении R C , как показано на рис.4.7 (б). Поскольку здесь нас интересуют только внешних сил , действующих на балку, мы можем рассматривать балку как твердое тело. Таким образом, луч вращается вокруг A, так что C перемещается в C ‘, а B перемещается в B’. Из подобных треугольников. мы видим, что

(i) Δv, B = aa + bΔv, C = aLΔv, C

Общая виртуальная работа, W t , выполненная всеми силами, действующими на балку, равна

(ii) Wt = RCΔv, C-WΔv, B

Обратите внимание, что работа, выполняемая нагрузкой, W , отрицательна, поскольку Δ v , B находится в направлении, противоположном его линии действия. .Также обратите внимание, что опорная реакция, R A , не работает, так как луч вращается только вокруг A. Теперь заменив Δ v , B в уравнении. (ii) из уравнения. (i) имеем

(iii) Wt = RCΔv, C − WaLΔv, C

Поскольку луч находится в равновесии, W t равно нулю из принципа виртуальной работы. Следовательно, из уравнения. (iii),

RCΔv, C — WaLΔv, C = 0

, что дает

RC = WaL

, что является результатом, который был бы получен при рассмотрении моментного равновесия балки относительно A.Определение R A выполняется аналогичным образом. Предположим теперь, что вместо одиночного смещения Δ v , C , всей балке задано виртуальное вертикальное смещение Δ v вместе с виртуальным вращением θ v , примерно A , как показано на Рис. 4.7 (c). Общая виртуальная работа, W t , выполненная силами, действующими на балку, теперь определяется выражением

(iv) Wt = RAΔv-W (Δv + aθv) + RC (Δv + Lθv) = 0

так как балка находится в равновесии.Переставляя уравнение. (iv),

(v) (RA + RC-W) Δv + (RCL-Wa) θv = 0

Уравнение (v) действительно для всех значений Δ v и θ v , поэтому что

RA + RC-W = 0, RCL-Wa = 0

, которые являются уравнениями равновесия, которые мы получили бы, решая силы по вертикали и принимая моменты около A.

1.3: Равновесные структуры, опорные реакции, определенность и Устойчивость балок и рам

Глава 3

Равновесные конструкции, реакции опор, определение и устойчивость балок и рам

3.1 Равновесие конструкций

Инженерные конструкции должны оставаться в равновесии как снаружи, так и внутри, когда подвергаются воздействию системы сил. Требования к равновесию для конструкций в двух и трех измерениях изложены ниже.

3.1.1 Равновесие в двух измерениях

Для того, чтобы конструкция, на которую действует система сил и пар, лежащих в плоскости xy , оставалась в состоянии покоя, она должна удовлетворять следующим трем условиям равновесия:

Три вышеуказанных условия обычно называют уравнениями равновесия для плоских конструкций.∑ F x и ∑ F y — это сумма составляющих x и y всех сил, действующих на конструкцию, а ∑ M z — сумма составляющих пара моментов и моментов всех сил относительно оси z , перпендикулярной плоскости xy действия сил.

3.1.2 Трехмерное равновесие

Трехмерная конструкция, то есть в пространстве, должна удовлетворять следующим шести требованиям, чтобы оставаться в равновесии при воздействии внешних сил:

3.2 типа опор и их характеристики

Тип опоры конструкции важен для обеспечения ее устойчивости. Опоры соединяют элемент с землей или с некоторыми другими частями конструкции. Предполагается, что студент уже знаком с несколькими типами опор для твердых тел, поскольку это было введено в курсе статики. Однако характеристики некоторых опор описаны ниже и показаны в Таблице 3.1.

3.2.1 Опора штифта или шарнира

Опора для штифта позволяет вращаться вокруг любой оси, но предотвращает перемещение в горизонтальном и вертикальном направлениях.Его идеализированное представление и реакции показаны в таблице 3.1.

3.2.2 Роликовая опора

Роликовая опора допускает вращение вокруг любой оси и поступательное движение (горизонтальное движение) в любом направлении, параллельном поверхности, на которую она опирается. Он удерживает конструкцию от движения в вертикальном направлении. Идеализированное изображение ролика и его реакции также показано в Таблице 3.1.

3.2.3 Коромысло

Характеристики коромысла аналогичны характеристикам роликовой опоры.Его идеализированная форма изображена в таблице 3.1.

3.2.4 Ссылка

У звена есть две петли, по одной с каждой стороны. Он допускает движение во всех направлениях, кроме направления, параллельного его продольной оси, которая проходит через два шарнира. Другими словами, сила реакции звена действует в направлении звена вдоль его продольной оси.

3.2.5 Фиксированная опора

Фиксированная опора ограничивает вращение в любом направлении и предотвращает перемещение как в горизонтальном, так и в вертикальном направлениях.

3.3 Определение и устойчивость балок и рам

Перед выбором аналитического метода важно установить детерминированность и стабильность конструкции. Определенная структура — это структура, неизвестная внешняя реакция или внутренние элементы которой могут быть определены с использованием только условий равновесия. Неопределенная конструкция — это такая конструкция, неизвестные силы которой не могут быть определены только условиями статического равновесия, и для ее полного анализа потребуется, кроме того, рассмотрение условий совместимости различных частей конструкции.Кроме того, конструкции должны быть стабильными, чтобы выполнять свои желаемые функции. Конструкция считается устойчивой, если она сохраняет свою геометрическую форму под действием внешних сил.

3.3.1 Формулировки для определения устойчивости и определения балок и рам

Условия определенности, неопределенности и нестабильности балок и рам можно сформулировать следующим образом:

где

r = количество опорных реакций.

C = уравнения состояния (два уравнения для одного внутреннего ролика и одно уравнение для каждого внутреннего пальца).

м = количество стержней.

j = количество соединений.

Таблица 3.1. Виды опор.

3.3.2 Альтернативная формулировка определения и устойчивости балок и рам

где

r = количество опорных реакций.

F i = количество сил реакции, передаваемых внутренним шарниром или внутренним роликом.

м = количество стержней.

Пример 3.1

Классифицируйте лучи, показанные на рисунках с 3.1 по 3.5, как стабильные, определенные или неопределенные, и укажите степень неопределенности там, где это необходимо.

Рис. 3.1. Луч.

Решение

Сначала нарисуйте диаграмму свободного тела каждой балки. Для определения классификации используйте уравнение 3.3 или уравнение 3.4.

Используя уравнение 3.3, r = 7, м = 2, c = 0, j = 3.Применение уравнения приводит к 3 (2) + 7> 3 (3) + 0 или 13> 9. Следовательно, луч статически неопределен до 4 °.

Используя уравнение 3.4, r = 7, m = 1, F i = 0. Применение уравнения приводит к 7 + 0> (3) (1) или 7> 3. Следовательно , луч статически неопределенен до 4 °.

Примечание. При использовании уравнения 3.3 части по обе стороны от внутренней опоры считаются отдельными элементами.

Рис. 3.2. Луч.

Решение

Используя уравнение 3.3, r = 6, m = 3, c = 0, j = 4. Применение уравнения приводит к 3 (3) + 6> 3 (4) + 0 или 15 > 12. Следовательно, луч статически неопределенен до 3 °.

Используя уравнение 3.4, r = 6, m = 1, F i = 0. Применение уравнения приводит к 6 + 0> (3) (1) или 6> 3.Следовательно, луч статически неопределенен до 3 °.

Рис. 3.3. Луч.

Решение

Используя уравнение 3.3, r = 5, m = 3, c = 1, j = 4. Применение уравнения приводит к 3 (3) + 5> 3 (4) + 1, или 14 > 13. Следовательно, луч статически неопределенен с точностью до 1 °.

Используя уравнение 3.4, r = 5, m = 2, F i = 2. Применение уравнения приводит к 5 + 2> 3 (2) или 7> 6.Следовательно, луч статически неопределенен с точностью до 1 °.

Рис. 3.4. Луч.

Решение

Используя уравнение 3.3, r = 5, m = 4, c = 1, j = 5. Применение уравнения приводит к 3 (4) + 5> 3 (5) + 1, или 17 > 16. Следовательно, уравнение статически неопределимо с точностью до 1 °.

Используя уравнение 3.4, r = 5, м = 2, F i = 2.Применение уравнения приводит к 5 + 2> 3 (2) или 7> 6. Следовательно, луч статически неопределен до 1 °.

Рис. 3.5. Луч.

Решение

Используя уравнение 3.3, r = 5, m = 5, c = 2, j = 6. Применение уравнения приводит к 3 (5) + 5 = 3 (6) + 2 или 20 = 20. Следовательно, балка статически определима.

Используя уравнение 3.4, r = 5, м = 3, F i = 4.Применение уравнения приводит к 5 + 4> 3 (3) или 9 = 9. Следовательно, луч статически определен.

Пример 3.2

Классифицируйте кадры, показанные на рисунках с 3.6 по 3.8, как стабильные или нестабильные, а также определяемые или неопределенные. Если неопределенность, укажите степень неопределенности.

Рис. 3.6. Рамка.

Решение

Используя уравнение 3.3, r = 3, м = 3, c = 0, j = 4.Применение уравнения приводит к 3 (3) + 3 = 3 (4) + 0 или 12 = 12. Таким образом, кадр является статически определенным.

Используя уравнение 3.4, r = 3, m = 1, F i = 0. Применение уравнения приводит к 3 + 0 = (3) (1) или 3 = 3. Следовательно , кадр статически определен.

Примечание. При использовании уравнения 3.3 для классификации рамы рама должна быть разобрана по соединениям, чтобы правильно определить количество элементов.

Рис. 3.7. Рамка.

Решение

Используя уравнение 3.3, r = 6, m = 3, c = 1, j = 4. Применение уравнения приводит к 3 (3) + 6> 3 (4) + 1, или 15 > 13. Следовательно, рамка статически неопределима до 2 °.

Используя уравнение 3.4, r = 6, m = 2, F i = 2. Применение уравнения приводит к 6 + 2> 3 (2) или 8> 6.Следовательно, рамка статически неопределима до 2 °.

Рис. 3.8. Рамка.

Решение

Используя уравнение 3.3, r = 4, m = 9, c = 0, j = 8. Применение уравнения приводит к 3 (9) + 4> 3 (8) + 0 или 31 > 24. Следовательно, рамка статически неопределима до 7 °.

Используя уравнение 3.4, r = 4, м = 1, F i = 9.Применение уравнения приводит к 4 + 9> (3) (2) или 13> 6. Следовательно, рамка статически неопределима до 7 °.

Примечание: При использовании уравнения 3.4 для классификации рамы с замкнутым контуром, как указано здесь, контур должен быть разрезан методом сечения, а внутренние реакции в разрезе должны учитываться при анализе.

3.4 Расчет опорных реакций для плоских конструкций

Опорные реакции для статически определенных и устойчивых конструкций на плоскости определяются с помощью уравнений равновесия.Процедура расчета описана ниже.

Порядок расчета реакций поддержки

• Нарисуйте диаграмму структуры в виде свободного тела, указав все неизвестные реакции с помощью диаграммы со стрелками.

• Проверьте устойчивость и определимость конструкции с помощью уравнения 3.3 или 3.4. Если структура классифицируется как детерминированная, приступайте к анализу.

• Определите неизвестные реакции, применив три уравнения равновесия. Если рассчитанная реакция приводит к отрицательному ответу, то первоначально предполагаемое направление неизвестной реакции, указанное стрелкой на диаграмме свободного тела, неверно и должно быть исправлено, чтобы показать противоположное направление.После внесения коррекции величина силы должна быть указана как положительное число в исправленном острие стрелки на диаграмме свободного тела

.

Пример 3.3

Консольная балка подвергается равномерно распределенной нагрузке и наклонной сосредоточенной нагрузке, как показано на рисунке 3.9a. Определите реакции на опоре A .

Рис. 3.9. Балка

Решение

Схема свободного тела. Схема свободного тела всей балки показана на рисунке 3.9b. Реакции опоры, как показано на диаграмме свободного тела, составляют A y , A x и M .

Расчет реакций. Перед расчетом опорных реакций распределенную нагрузку следует заменить единственной равнодействующей силой, а наклонную нагрузку разделить на вертикальную и горизонтальную составляющие. Величина результирующей силы равна площади прямоугольной нагрузки, и она действует через центр тяжести прямоугольника.Как видно на рисунке 3.9c, P = [(4 кН / м) (2 м)], и его положение находится в центре тяжести прямоугольной нагрузки. Применение уравнений статического равновесия дает следующее:

Пример 3.4

Простая балка длиной 12 футов несет равномерно распределенную нагрузку 2 тысячи фунтов / фут по всему пролету и сосредоточенную нагрузку 8 тысяч фунтов в середине пролета, как показано на рисунке 3.10a. Определите реакции на опорах A и B балки.

Рис. 3.10. Простая балка.

Решение

Схема свободного тела. Схема свободного тела всей балки показана на рисунке 3.10b.

Расчет реакций. Распределенная нагрузка сначала заменяется единственной равнодействующей силой, как показано на рисунке 3.10c. Величина результирующей силы равна площади прямоугольного нагружения (распределенной силы). Таким образом, P = [(2 k / ft) (12 ft)], и его местоположение находится в центре тяжести прямоугольной нагрузки. Поскольку в этом примере существует симметрия нагрузки, реакции на обоих концах балки равны, и они могут быть определены с использованием уравнений статического равновесия и принципа суперпозиции следующим образом:

Пример 3.5

Балка с выступом подвергается переменной нагрузке, как показано на рисунке 3.11a. Определите реакции на опорах A и B .

Рис. 3.11. Балка с вылетом.

Решение

Схема свободного тела. Схема свободного тела всей балки показана на рис. 3.11b.

Расчет реакций. Обратите внимание, что распределенная нагрузка в балке имеет треугольную форму. Распределенная нагрузка сначала заменяется единственной равнодействующей силой, как показано на рисунке 3.11c. Величина единственной равнодействующей силы равна площади под треугольной нагрузкой. Таким образом, P = () (6 м) (10 кН / м), а его центр тяжести находится в центре нагрузки (6 м). Применение уравнений равновесия дает следующее:

Пример 3.6

Балка с выступающими концами выдерживает три сосредоточенных нагрузки в 12 тысяч фунтов, 14 тысяч фунтов и 16 тысяч фунтов и момент 100 тысяч фунтов на фут, как показано на рисунке 3.12a. Определите реакции на опорах A и B .

Рис. 3.12. Балка с свисающими концами.

Решение

Схема свободного тела. Схема балки со свободным телом показана на рис. 3.12b.

Расчет реакций. Применение уравнений равновесия дает следующее:

Пример 3,7

Составная балка подвергается нагрузкам, показанным на рисунке 3.13a. Найдите реакции опоры на A и B балки.

Рис.3.13. Составная балка.

Решение

Схема свободного тела. Схема свободного тела всей балки показана на рис. 3.13b.

Идентификация первичных и дополнительных структур. Для правильного анализа составной структуры необходимо определить первичную и дополнительную части структуры для правильного понимания их взаимодействия. Взаимодействие этих частей показано на рисунке 3.13c. Первичная структура — это часть составной конструкции, которая может выдерживать приложенную внешнюю нагрузку без помощи дополнительной конструкции.С другой стороны, дополнительная структура — это часть составной конструкции, которая зависит от первичной конструкции, чтобы выдерживать приложенную внешнюю нагрузку. Для данной структуры часть AC является первичной структурой, а часть CB — дополнительной структурой.

Расчет реакций. Анализ составной структуры всегда должен начинаться с анализа дополнительной структуры, поскольку дополнительная структура поддерживается первичной структурой.С помощью уравнений равновесия опорные реакции балки определяются следующим образом:

Анализ дополнительной структуры CB.

Расчет опорной реакции. Схема изолированного свободного тела дополнительной конструкции показана на рис. 3.13c. Во-первых, распределенная нагрузка заменяется единственной равнодействующей силой ( P ), которая равна площади прямоугольной нагрузки, как показано на рисунках 3.13d и 3.13e. Применяя уравнения равновесия и учитывая, что из-за симметрии нагрузки, реакции опоры в точке C и B равны по величине, получаем следующее:

Анализ первичной структуры AC.

Расчет опорной реакции. Обратите внимание, что перед вычислением реакций реакция в точке C в дополнительной структуре применяется к первичной структуре в качестве нагрузки.Величина прилагаемой нагрузки такая же, как у дополнительной конструкции, но противоположная по направлению. Применение уравнений равновесия предполагает следующее:

Пример 3.8

Найдите реакции на опорах A , C и E составной балки, несущей равномерно распределенную нагрузку 10 тысяч фунтов / фут по всей ее длине, как показано на рисунке 3.14a.

Рис.3.14. Составная балка.

Решение

Схема свободного тела. Схема свободного тела всей балки показана на рис. 3.14b.

Идентификация первичных и дополнительных структур. Схема взаимодействия данной структуры показана на рисунке 3.14c. AB является первичной структурой, а BD и DE являются дополнительными структурами.

Расчет реакций.

Анализ комплементарной структуры DE.

Расчет опорной реакции. Схема изолированного свободного тела показана на рис. 3.14c. Во-первых, распределенная нагрузка заменяется единственной равнодействующей силой ( P ), равной площади прямоугольной нагрузки, как показано на рисунке 3.14d. Применяя уравнения равновесия и отмечая, что из-за симметрии нагрузки, реакции опоры в точке D и E равны по величине, позволяет предположить следующее:

Анализ комплементарной структуры БД.

Расчет опорной реакции. Схема изолированного свободного тела показана на рис. 3.14e. Во-первых, распределенная нагрузка заменяется единственной равнодействующей силой ( P ), равной площади прямоугольной нагрузки, как показано на рисунке 3.14f. Нагрузка от дополнительной конструкции приложена в точке D . Применение уравнений равновесия предполагает следующее:

Анализ первичной структуры AB.

Расчет опорной реакции.Обратите внимание, что перед вычислением реакций равномерная нагрузка заменяется единственной равнодействующей силой, а реакция в точке B в дополнительной конструкции применяется к первичной конструкции в качестве нагрузки. Применение требования равновесия дает следующее:

Пример 3.9

Найдите реакции на опорах A , B , E и F нагруженной составной балки, как показано на рисунке 3.15a.

Рис. 3.15. Составная балка.

Решение

Схема свободного тела. Схема свободного тела всей балки показана на рис. 3.15b.

Идентификация первичной и дополнительной структуры. Схема взаимодействия данной структуры показана на рисунке 3.15c. CD является дополнительной структурой, а AC и DF являются первичными структурами.

Расчет реакций.

Анализ комплементарной структуры CD.

Расчет опорной реакции. Схема изолированного свободного тела показана на рисунке 3.15c. Во-первых, распределенная нагрузка заменяется единственной равнодействующей силой ( P ), которая равна площади прямоугольной нагрузки, как показано на рисунке 3.15d. Применяя уравнения равновесия и отмечая, что из-за симметрии нагрузки, реакции опоры в точке C и D равны по величине, позволяет предположить следующее:

Анализ первичной структуры AC.

Расчет опорной реакции. Обратите внимание, что реакция на C дополнительной структуры применяется как направленная вниз сила такой же величины в той же точке на первичной структуре. Применение уравнения равновесия предполагает следующее:

Анализ первичной структуры DF.

Расчет опорной реакции. Схема изолированного свободного тела показана на рисунке 3.15f. Во-первых, распределенная нагрузка заменяется единственной равнодействующей силой ( P ), равной площади треугольной нагрузки, как показано на рисунке 3.15g. Применяя уравнения равновесия и отмечая, что реакция опоры в точке D дополнительной конструкции действует как нагрузка на первичную конструкцию, позволяет предположить следующее:

Пример 3.10

Определите реакции на опорах A и D рамы, показанной на рисунке 3.16а.

Рис. 3.16. Рамка.

Решение

Схема свободного тела. Схема свободного тела всей балки показана на рис. 3.16b.

Расчет реакций. Распределенные нагрузки в колонне AB и балке BC сначала заменяются единичными равнодействующими силами, определяемыми как площадь их соответствующего оттенка нагрузки, как показано на рисунке 3.16c. Применение условий равновесия предполагает следующее:

Пример 3.11

Жесткая рама нагружается, как показано на Рисунке 3.17a. Определите реакции на опоре D .

Рис. 3.17. Жесткий каркас.

Решение

Схема свободного тела. Схема свободного тела всей балки показана на рис. 3.17b.

Расчет реакций. Распределенная нагрузка в части AB рамы сначала заменяется единственной равнодействующей силой, как показано на рисунке 3.17c. Применение уравнений равновесия предполагает следующее:

Пример 3.12

Найдите реакции на опорах E и F рамы, показанной на рисунке 3.18a.

Рис. 3.18. Рамка.

Решение

Схема свободного тела. Схема рамы со свободным телом показана на рис. 3.18b.

Расчет реакций. Распределенные нагрузки сначала заменяются одиночными равнодействующими силами, как показано на рисунке 3.18c. Применение уравнений статического равновесия предполагает следующее:

Пример 3.13

Определите реакции на опоре A жесткой рамы, показанной на Рисунке 3.19a.

Рис. 3.19. Жесткий каркас.

Решение

Схема свободного тела. Схема рамы со свободным телом показана на рис. 3.19b.

Расчет реакций. Распределенная нагрузка в колонне AB сначала заменяется единственной равнодействующей силой, как показано на рисунке 3.19c. Применение уравнений статического равновесия предполагает следующее:

Пример 3.14

Определите реакции на опорах A и E рамы, шарнирно закрепленной на C , как показано на рисунке 3.20a.

Рис. 3.20. Рамка.

Решение

Схема свободного тела. Схема рамы со свободным телом показана на рис. 3.20b.

Расчет реакций. Реакции в составном каркасе вычисляются с учетом диаграмм свободного тела как всего кадра, так и его части.Перед расчетом реакций распределенная нагрузка в колонне заменяется единственной равнодействующей силой. Вертикальные реакции на E и A и горизонтальные реакции на A находятся путем применения уравнений статического равновесия и рассмотрения диаграммы свободного тела всей рамы. Горизонтальная реакция в точке E находится при рассмотрении части CDE диаграммы свободного тела.

Отрицательный знак означает, что первоначально предполагаемое направление A y было неправильным.Следовательно, A y действует вниз, а не вверх, как предполагалось изначально. Это следует исправить в последующем анализе.

Чтобы определить E x , рассмотрите момент силы в элементе CDE относительно шарнира.

Пример 3.15

Найдите реакции на опоре A и B нагруженной рамы на рис. 3.21a. Рама навесная на D .

Рис. 3.21. Загруженный кадр.

Решение

Схема свободного тела. Схема рамы со свободным телом показана на рис. 3.21b.

Расчет реакций. Распределенная нагрузка в столбце AC сначала заменяется одной равнодействующей силой путем определения области нагрузки, как показано на рис. 3.21Figurec. Реакция на B вычисляется, принимая момент сил в части DB рамы относительно штифта на D , а другие реакции определяют с применением других условий равновесия.

Отрицательный знак означает, что первоначально предполагаемое направление A y было неправильным. Следовательно, A y действует вниз, а не вверх, как предполагалось изначально. Это следует исправить в последующем анализе.

Краткое содержание главы

Условия статического равновесия: Конструкция находится в состоянии статического равновесия, если равнодействующая всех сил и моментов, действующих на нее, равна нулю.Математически это выражается следующим образом:

F = 0∑ M = 0

Для тела в плоскости существуют следующие три уравнения равновесия:

F x = 0∑ F y = 0∑ M 0 = 0

Типы опор: Различные символьные представления используются для моделирования различных типов опор для конструкций. Ролик используется для моделирования опоры, которая предотвращает вертикальное движение конструкции, но допускает горизонтальное перемещение и вращение.Штифт используется для моделирования опоры, которая предотвращает горизонтальные и вертикальные движения, но допускает вращение. Фиксированная опора моделирует опору, предотвращающую горизонтальные и вертикальные движения и вращение.

Детерминированность, неопределенность и стабильность структур: Структура определена, если количество неизвестных реакций равно количеству статического равновесия. Таким образом, уравнений статического равновесия достаточно для определения опор такой конструкции.С другой стороны, статически неопределенная структура — это структура, в которой количество неизвестных реакций превышает количество уравнений равновесия. Для анализа неопределенной конструкции необходимы дополнительные уравнения, и эти уравнения могут быть получены с учетом совместимости конструкции. Неопределенные конструкции иногда необходимы, когда необходимо уменьшить размеры элементов или увеличить их жесткость. Стабильная структура — это структура, в которой реакции поддержки не параллельны и не совпадают друг с другом.Формулировки устойчивости и определенности балок и рам следующие:

Балки и рамы:

3 м + r <3 j + C Структура нестабильна

3 м + r = 3 j + C Структура определенная

3 м + r > 3 j + C Структура неопределенная

Практические задачи

3.1 Классифицируйте конструкции, показанные на рисунках P3.1a – P3.1p, как статически определенные или неопределенные, а также статически устойчивые или нестабильные. Если неопределенность, укажите степень неопределенности.

Рис. P3.1. Классификация конструкций.

3.2. Определите опорные реакции для балок, показанных на рисунках с P3.2 по P3.12.

Рис. P3.2. Луч.

Рис. P3.3. Луч.

Рис.P3.4. Луч.

Рис. P3.5. Луч.

Рис. P3.6. Луч.

Рис. P3.7. Луч.

Рис. P3.8. Луч.

Рис. P3.9. Луч.

Рис. P3.10. Луч.

Рис. P3.11. Луч.

Рис. P3.12. Луч.

3.3. Определите реакции опоры для рам, показанных на рисунках с P3.13 по P3.20.

Рис.P3.13. Рамка.

Рис. P3.14. Рамка.

Рис. P3.15. Рамка.

Рис. P3.16. Рамка.

Рис. 3.17. Рамка.

Рис. 3.18. Рамка.

Рис. 3.19. Рамка.

Рис. 3.20. Рамка.

3.4 Определите опорные реакции для ферм, показанных на рисунках с P3.21 по P3.27.

Рис. P3.21. Ферма.

Рис. P3.22. Ферма.

Рис. P3.23. Ферма.

Рис. P3.24. Ферма.

Рис. P3.25. Ферма.

Рис. P3.26. Ферма.

Рис. P3.27. Ферма.

Лучевые реакции и диаграммы — Приложение к сопротивлению материалов для энергетики

Диаграммы

Цели обучения

В конце этой главы вы должны уметь:

  • Определение реакции свободно опертых, выступающих и консольных балок
  • Рассчитайте и нарисуйте диаграммы силы сдвига и изгибающего момента балок, подверженных сосредоточенным нагрузкам, равномерно распределенным нагрузкам и их комбинациям.

Балки обзор

Балки — это конструкционные элементы для различных инженерных применений, таких как крыши, мосты, механические узлы и т. Д. В целом балки являются тонкими, прямыми, жесткими, изготовлены из изотропных материалов и, что наиболее важно, подвергаются нагрузкам, перпендикулярным их продольной оси. Если вместо перпендикулярных нагрузок тот же элемент конструкции будет подвергаться продольным нагрузкам, он будет называться колонной или стойкой. Если тот же самый элемент будет подвергаться крутящему моменту, он будет называться и рассматриваться как вал.Поэтому при определении механических или конструктивных компонентов очень важно учитывать способ нагрузки.

Обратите внимание, что когда дело доходит до ориентации, балки могут быть горизонтальными, вертикальными или с любым наклоном между ними (например, погруженные пластины, анализируемые в гидромеханике)… при условии, что нагрузка перпендикулярна их главной оси.

Опоры балок:

Нагрузки на балку :

Нагрузки Обозначение Примеры Покрытый
Точка, также называемая
  • колеса автомобиля
  • столбца
  • человек на трамплине
Есть
Равномерное распределенное
  • вес балки
  • снеговая нагрузка на ферму крыши
Есть
Переменная Распределенная
  • гидростатическая нагрузка на подводную поверхность
  • свая из заполнителя
  • балка переменного сечения
Есть
Концентрированные моменты

Типы балок:

Решение для лучевых реакций

При решении для реакций рекомендуются следующие шаги:

  1. Нарисуйте диаграмму тела без балки
  2. Замените равномерно распределенную нагрузку (если есть) эквивалентной точечной нагрузкой
  3. Решите ΣM A = 0 (сумма моментов относительно опоры A).Это даст вам B рэндов (реакция на поддержку B).
  4. Решите ΣM B = 0. Это даст вам R A .
  5. Используя R A и R B , найденный на шагах 3 и 4, проверьте, удовлетворяется ли ΣV = 0 (сумма всех вертикальных сил).
    1. Обратите внимание, что шаги 4 и 5 можно поменять местами.
    2. Для консольной балки используйте ΣV = 0, чтобы найти вертикальную реакцию на стене, и ΣM wall = 0, чтобы найти моментную реакцию на стене. Другого уравнения для подтверждения ваших результатов нет.

Диаграммы поперечных сил и изгибающих моментов

Обратите внимание:

«Сдвиговые силы — это внутренние силы, развивающиеся в материале балки для уравновешивания приложенных извне сил для обеспечения равновесия всех частей балки.

Изгибающие моменты — это внутренние моменты, возникающие в материале балки для уравновешивания тенденции внешних сил вызывать вращение любой части балки ». [3]

Сила сдвига в любом сечении балки может быть найдена путем суммирования всех вертикальных сил слева или справа от рассматриваемого сечения.

Точно так же изгибающий момент в любом сечении балки можно найти, сложив моменты слева или справа от рассматриваемого сечения. Опорной точкой момента является рассматриваемое место.

По соглашению внутренние сдвигающие силы, действующие вниз, считаются положительными. Они противодействуют восходящим внешним силам. Следовательно, при представлении поперечных сил вы можете нарисовать их в направлении внешних сил. Это визуально проще, чем следовать условным обозначениям.

Моменты по часовой стрелке обычно считаются отрицательными, а моменты против часовой стрелки — положительными. При представлении изменения изгибающего момента обратитесь к следующей таблице, в которой показаны качественные кривые изгибающего момента в зависимости от формы графиков поперечной силы.

.

При построении диаграмм поперечных сил и изгибающих моментов важны условные обозначения, но решающее значение имеет согласованность. Например, рассмотрим простую балку, нагруженную точечной нагрузкой, приложенной к нагрузке UD.Запуск диаграмм на опоре A, глядя на страницу, сгенерирует следующее:

Теперь переверните луч горизонтально на 180 ° (или измените точку наблюдения, глядя на луч с противоположной стороны) и начертите диаграммы, начиная с той же точки A. Диаграммы будут выглядеть следующим образом:

Обратите внимание, что, хотя диаграммы поперечных сил выглядят как зеркальные изображения (перевернутые по горизонтали), на диаграмму изгибающего момента это не влияет. Кроме того, наиболее важный результат этого анализа показывает, что значения максимальной силы сдвига и изгибающего момента всегда будут одинаковыми.

КПП для схемы балок

При построении схем балок необходимо учитывать следующее:

Диаграммы поперечных сил:

  • На концах свободно опертой балки поперечная сила равна нулю.
  • У стены консольной балки поперечная сила равна вертикальной реакции у стены. На свободном конце балки поперечная сила равна нулю.
  • На любом сегменте балки, где отсутствуют нагрузки, поперечная сила остается постоянной (горизонтальная линия).
  • Точечная нагрузка или реакция на диаграмме поперечных сил приводит к резкому изменению диаграммы в направлении приложенной нагрузки.
  • Равномерно распределенная нагрузка, действующая на балку, представлена ​​прямой поперечной силой с отрицательным или положительным наклоном, равной нагрузке на единицу длины.

Диаграмма изгибающих моментов:

  • На концах свободно опертой балки изгибающие моменты равны нулю.
  • У стенки консольной балки изгибающий момент равен моменту реакции.На свободном конце изгибающий момент равен нулю.
  • В том месте, где поперечная сила пересекает нулевую ось, соответствующий изгибающий момент имеет максимальное значение.
  • Форма кривой изгибающего момента между двумя точками балки показана в двух таблицах выше.
  • Изменение изгибающего момента между двумя точками балки равно площади под диаграммой поперечных сил между теми же двумя точками.

Приведенные выше рекомендации помогут вам в построении диаграмм направленности; они также служат проверкой.

Назначенные задачи

Рассчитайте реакции балки и нарисуйте диаграммы поперечной силы и изгибающего момента для следующих балок.

При решении диаграмм пучка в классе и дома вы можете проверить свои ответы с помощью бесплатного онлайн-калькулятора пучка: SkyCiv Cloud Engineering Software

Задача 1: Укажите максимальные значения поперечной силы и изгибающего момента.

Задача 2: Укажите максимальные значения поперечной силы и изгибающего момента.

Проблема 3: Балка длиной 24 метра просто опирается на 3 метра с каждого конца. Балка несет точечную нагрузку 18 кН на левом конце и 22 кН на правом конце балки. Балка весит 400 кг / м. Нарисуйте схемы балок и определите место на балке, где изгибающий момент равен нулю.

Задача 4: Простая свисающая балка длиной 112 футов выступает над левой опорой на 14 футов. Балка несет сосредоточенную нагрузку в 90 тысяч фунтов на 12 футов от правого конца и равномерно распределенную нагрузку в 12 тысяч фунтов / фут на 40 футов. раздел с левого конца.Нарисуйте схемы балок и определите поперечную силу и изгибающий момент на участке в 50 футах от левого конца.

Задача 5: Предложите улучшение для этой главы.

Калькулятор нормальной силы

| Плоская и наклонная поверхность

Калькулятор нормальной силы поможет вам найти силу, которую оказывает поверхность, чтобы предотвратить падение предмета через нее. В следующем тексте мы предоставим вам некоторые формулы нормальной силы и ответ на простой вопрос: что такое нормальная сила? При этом мы также упомянем третий закон движения Ньютона.Прокрутите вниз, чтобы узнать, как рассчитать нормальную силу.

Раз уж вы здесь, возможно, вам понравится наш калькулятор второго закона Ньютона.

Что такое нормальная сила?

Нормальная сила — это перпендикулярная сила, которую поверхность оказывает на объект . Например, если вы положите книгу на стол, ее притянет к земле гравитационная сила. Чтобы противодействовать этой силе, стол оказывает давление на книгу, предотвращая ее падение. Эта противодействующая сила называется нормальной силой и представлена ​​как F N или N .Единица измерения нормальной силы — « Н » (Ньютон).

Нормальная сила — типичный пример третьего закона движения Ньютона.

Если один объект оказывает силу на второй объект, второй объект оказывает на первый объект силу равной величины и в противоположном направлении (действие равно противодействию).

Итак, нормальная сила равна силе, прилагаемой объектом к поверхности. Его формулы меняются в зависимости от уклона поверхности.

  1. Для объекта, лежащего на плоской поверхности, формула:

F N = m * g

где

  • м — масса объекта.
  • г — ускорение свободного падения.

Согласно третьему закону Ньютона нормальная сила ( F N ) для объекта на плоских поверхностях равна его гравитационной силе ( W ).

  1. Для объекта, расположенного на наклонной поверхности, уравнение нормальной силы имеет следующий вид:

F N = m * g * cos (α)

где

  • α — угол наклона поверхности.

На наклонной поверхности (при условии, что объект не скользит вниз) вес объекта поддерживается как нормальной силой, так и трением. Сила тяжести объекта не противоположна и равна нормальной силе, но одна из составляющих вектора силы тяжести равна.

Чтобы узнать, как рассчитать нормальную силу с трением, воспользуйтесь калькулятором трения.

Для предметов на плоской поверхности нормальная сила противодействует весу предметов .(Не путайте это с массой! Вес — это то же самое, что и сила тяжести.) Это только в том случае, когда на объект нет внешней силы, или, если она есть, внешняя сила параллельна поверхности. Давайте посмотрим, что произойдет, если есть внешняя сила, которая не действует одновременно с поверхностью!

Формула нормальной силы с внешней силой

При расчетах с учетом внешней силы следует учитывать только параллельную составляющую вектора. Вот почему в уравнения нормальной силы, перечисленные ниже, включены углы.

  1. Внешняя направленная вниз сила

F N = m * g + F * sin (x)

где

  • F — значение внешней силы.
  • x — угол между поверхностью и внешней силой.
  1. Внешняя направленная вверх сила

F N = m * g - F * sin (x)

Если есть внешняя сила, направленная вниз, вам нужно добавить ее векторную составляющую к весу объекта.Это увеличивает нормальную силу, внешняя сила толкает объект в землю. Противоположный случай для внешней силы, направленной вверх. Он отталкивает объект от земли, поэтому нормальная сила уменьшается.

Если сила направлена ​​прямо вверх и равна силе тяжести, нормальная сила равна нулю. Почему? Потому что он полностью противодействует силе тяжести.

Как использовать уравнение нормальной силы — пример

Как самостоятельно найти нормальную силу? Представьте, что на земле лежит ящик, который вы хотите переместить.Его вес составляет 100 кг. Вы нажимаете на него под углом 45 градусов с силой 250 Н.

N = 100 * 9,807 + 250 * sin (45 o ) = 980,7 + 250 * √2 / 2 = 1,157,4 N

Земля оказывает на коробку усилие 1,157,4 Н . Если коробка находится на мягкой поверхности, из-за вашей дополнительной силы она может рухнуть. Таким образом, чтобы сберечь силы, лучше толкать вещи рядом, прямо к месту назначения.

Вам понравился наш калькулятор нормальной силы? Также ознакомьтесь с калькулятором работы и мощности!

12.2 Примеры статического равновесия

Цели обучения

К концу этого раздела вы сможете:

  • Выявление и анализ ситуаций статического равновесия
  • Построение диаграммы свободного тела для протяженного объекта в статическом равновесии
  • Установка и решение условий статического равновесия для объектов, находящихся в равновесии, в различных физических ситуациях

Все примеры в этой главе относятся к планарным задачам. Соответственно, мы используем условия равновесия в компонентной форме от (Рисунок) до (Рисунок).Мы ввели стратегию решения проблем на (Рисунок), чтобы проиллюстрировать физический смысл условий равновесия. Теперь мы обобщим эту стратегию в виде списка шагов, которые необходимо соблюдать при решении задач статического равновесия для протяженных твердых тел. Мы выполняем пять практических шагов.

Стратегия решения проблем: статическое равновесие
  1. Укажите объект для анализа. Для некоторых систем, находящихся в равновесии, может потребоваться рассмотреть более одного объекта. Определите все силы, действующие на объект.Определите вопросы, на которые вам нужно ответить. Определите информацию, содержащуюся в проблеме. В реальных задачах некоторая ключевая информация может быть скрыта в ситуации, а не предоставлена ​​явно.
  2. Создайте диаграмму свободного тела для объекта. (a) Выберите для задачи справочную рамку xy . Нарисуйте для объекта диаграмму свободного тела, включая только действующие на него силы. Если возможно, представьте силы в виде их компонентов в выбранной системе отсчета.Когда вы делаете это для каждой силы, вычеркните исходную силу, чтобы ошибочно не включить одну и ту же силу в уравнения. Обозначьте все силы — это понадобится вам для правильного расчета чистых сил в направлениях x и y . Для неизвестной силы направление должно быть задано произвольно; думайте об этом как о «рабочем направлении» или «предполагаемом направлении». Правильное направление определяется знаком, который вы получаете в окончательном решении. Знак плюс [латекс] (+) [/ латекс] означает, что рабочее направление является фактическим направлением.Знак минус [латекс] (-) [/ латекс] означает, что фактическое направление противоположно предполагаемому рабочему направлению. (б) Выберите положение оси вращения; Другими словами, выберите точку поворота, относительно которой вы будете вычислять моменты действующих сил. На схеме свободного тела укажите расположение оси и плеч рычага действующих сил — это понадобится вам для правильного расчета крутящих моментов. При выборе шарнира имейте в виду, что шарнир можно разместить где угодно, но руководящий принцип заключается в том, что лучший выбор максимально упростит расчет чистого крутящего момента вдоль оси вращения.
  3. Составьте уравнения равновесия для объекта. (a) Используйте диаграмму свободного тела, чтобы записать правильное состояние равновесия (рисунок) для компонентов силы в направлении x . (b) Используйте диаграмму свободного тела, чтобы записать правильное состояние равновесия (рисунок) для компонентов силы в направлении y . (c) Используйте диаграмму свободного тела, чтобы записать правильное состояние равновесия (рисунок) для крутящих моментов вдоль оси вращения. Используйте (Рисунок) для оценки величин и чувствительности крутящего момента.
  4. Упростите и решите систему уравнений равновесия, чтобы получить неизвестные величины. На данный момент ваша работа связана только с алгеброй. Имейте в виду, что количество уравнений должно быть таким же, как и количество неизвестных. Если количество неизвестных больше, чем количество уравнений, проблема не может быть решена.
  5. Оцените выражения для неизвестных величин, которые вы получили в своем решении. В ваших окончательных ответах должны быть правильные числовые значения и правильные физические единицы.В противном случае используйте предыдущие шаги, чтобы отследить ошибку до ее источника и исправить ее. Кроме того, вы можете самостоятельно проверить свои числовые ответы, переместив точку поворота в другое место и снова решив проблему, что мы и сделали на (рисунок).

Обратите внимание, что построение диаграммы свободного тела для задачи равновесия твердого тела является наиболее важным компонентом в процессе решения. Без правильной настройки и правильной диаграммы вы не сможете записать правильные условия равновесия.Также обратите внимание, что диаграмма свободного тела для протяженного твердого тела, которое может совершать вращательное движение, отличается от диаграммы свободного тела для тела, которое испытывает только поступательное движение (как вы видели в главах, посвященных законам движения Ньютона). В поступательной динамике тело представляется как его ЦМ, в котором все силы прилагаются к телу, а крутящие моменты отсутствуют. Это не относится к динамике вращения, где протяженное твердое тело не может быть представлено одной точкой. Причина этого в том, что при анализе вращения мы должны идентифицировать крутящие моменты, действующие на тело, а крутящий момент зависит как от действующей силы, так и от плеча рычага.Здесь диаграмма свободного тела для протяженного твердого тела помогает нам определить внешние моменты.

Пример

Балансировка крутящего момента

Три гири прикреплены к единой измерительной линейке, как показано на (Рисунок). Масса измерительного стержня составляет 150,0 г, а массы слева от точки опоры составляют [латекс] {m} _ {1} = 50,0 \, \ text {g} [/ latex] и [латекс] {m} _ {2} = 75,0 \, \ text {g}. [/ latex] Найдите массу [латекс] {m} _ {3} [/ latex], которая уравновешивает систему, когда она прикреплена к правому концу ручки, и нормальную силу реакции в точке опоры, когда система уравновешена .

Рисунок 12.9 При балансировке крутящего момента горизонтальная балка опирается на точку опоры (обозначена буквой S), а массы прикрепляются к обеим сторонам оси. Система находится в статическом равновесии, когда балка не вращается. Он уравновешен, когда луч остается ровным.

Стратегия

Для схемы, показанной на рисунке, мы выделяем следующие пять сил, действующих на измерительную линейку:

[латекс] {w} _ {1} = {m} _ {1} г [/ латекс] — масса [латекса] {m} _ {1}; [/ latex] [latex] {w} _ {2} = {m} _ {2} g [/ latex] — масса [латекса] {m} _ {2}; [/ латекс]

[латекс] w = мг [/ латекс] — вес всей измерительной линейки; [латекс] {w} _ {3} = {m} _ {3} g [/ latex] — вес неизвестной массы [латекс] {m} _ {3}; [/ латекс]

[латекс] {F} _ {S} [/ латекс] — нормальная сила реакции в точке опоры S .

Мы выбираем систему отсчета, в которой направление оси y — это направление силы тяжести, направление оси x — вдоль измерительной ручки, а ось вращения (ось z ) ) перпендикулярна оси x и проходит через точку опоры S . Другими словами, мы выбираем ось в точке соприкосновения измерительной линейки с опорой. Это естественный выбор для поворота, потому что эта точка не перемещается при вращении ручки.Теперь мы готовы создать диаграмму свободного тела для измерительной ручки. Мы указываем ось и присоединяем пять векторов, представляющих пять сил, вдоль линии, представляющей стержень измерителя, располагая силы относительно оси (рисунок). На этом этапе мы можем идентифицировать рычаги пяти сил, учитывая информацию, предоставленную в задаче. Для трех висящих грузов проблема явно связана с их расположением вдоль стержня, но информация о расположении груза w дается неявно.Ключевое слово здесь — «униформа». Из наших предыдущих исследований мы знаем, что ЦМ однородной палки находится в ее средней точке, поэтому здесь мы прикрепляем груз w на отметке 50 см.

Рисунок 12.10 Схема свободного тела для измерительной ручки. Пивот выбирается в точке поддержки S.

Решение

Используя (Рисунок) и (Рисунок) для справки, мы начинаем с нахождения плеч рычагов пяти сил, действующих на палку:

[латекс] \ begin {array} {ccc} \ hfill {r} _ {1} & = \ hfill & 30.0 \, \ text {cm} +40.0 \, \ text {cm} = 70.0 \, \ text {cm} \ hfill \\ \ hfill {r} _ {2} & = \ hfill & 40.0 \, \ text { cm} \ hfill \\ \ hfill r & = \ hfill & 50.0 \, \ text {cm} -30.0 \, \ text {cm} = 20.0 \, \ text {cm} \ hfill \\ \ hfill {r} _ { S} & = \ hfill & 0.0 \, \ text {cm} \, \ text {(потому что} \, {F} _ {S} \, \ text {прикреплен к опоре)} \ hfill \\ \ hfill {r} _ {3} & = \ hfill & 30.0 \, \ text {см.} \ hfill \ end {array} [/ latex]

Теперь мы можем найти пять крутящих моментов относительно выбранной оси:

[латекс] \ begin {array} {ccccc} \ hfill {\ tau} _ {1} & = \ hfill & + {r} _ {1} {w} _ {1} \ text {sin} \, 90 \ text {°} = \ text {+} {r} _ {1} {m} _ {1} g \ hfill & & \ text {(вращение против часовой стрелки, положительное значение)} \ hfill \\ \ hfill {\ tau } _ {2} & = \ hfill & + {r} _ {2} {w} _ {2} \ text {sin} \, 90 \ text {°} = \ text {+} {r} _ {2 } {m} _ {2} g \ hfill & & \ text {(вращение против часовой стрелки, положительное значение)} \ hfill \\ \ hfill \ tau & = \ hfill & + rw \, \ text {sin} \, 90 \ text {°} = \ text {+} rmg \ hfill & & \ text {(гравитационный момент)} \ hfill \\ \ hfill {\ tau} _ {S} & = \ hfill & {r} _ {S} { F} _ {S} \ text {sin} \, {\ theta} _ {S} = 0 \ hfill & & \ text {(потому что} \, {r} _ {S} = 0 \, \ text {см )} \ hfill \\ \ hfill {\ tau} _ {3} & = \ hfill & \ text {-} {r} _ {3} {w} _ {3} \ text {sin} \, 90 \ text {°} = \ text {-} {r} _ {3} {m} _ {3} g \ hfill & & \ text {(вращение по часовой стрелке, отрицательное значение)} \ hfill \ end {array} [/ latex]

Второе условие равновесия (уравнение для крутящих моментов) для измерительной ручки —

[латекс] {\ tau} _ {1} + {\ tau} _ {2} + \ tau + {\ tau} _ {S} + {\ tau} _ {3} = 0.[/ латекс]

При подстановке значений крутящего момента в это уравнение мы можем опустить крутящие моменты, дающие нулевой вклад. Таким образом, второе условие равновесия —

.

[латекс] + {r} _ {1} {m} _ {1} g + {r} _ {2} {m} _ {2} g + rmg- {r} _ {3} {m} _ { 3} г = 0. [/ латекс]

Если выбрать направление [latex] + y [/ latex], параллельное [latex] {\ overset {\ to} {F}} _ {S}, [/ latex], первое условие равновесия для ручки —

[латекс] \ text {-} {w} _ {1} — {w} _ {2} -w + {F} _ {S} — {w} _ {3} = 0. [/ латекс]

Подставляя силы, первое условие равновесия становится

[латекс] \ text {-} {m} _ {1} g- {m} _ {2} g-mg + {F} _ {S} — {m} _ {3} g = 0.[/ латекс]

Мы решаем эти уравнения одновременно для неизвестных значений [латекс] {m} _ {3} [/ latex] и [латекс] {F} _ {S}. [/ latex] В (Рисунок) мы отменяем коэффициент g и переставляем члены, чтобы получить

[латекс] {r} _ {3} {m} _ {3} = {r} _ {1} {m} _ {1} + {r} _ {2} {m} _ {2} + rm . [/ латекс]

Чтобы получить [латекс] {m} _ {3} [/ latex], мы делим обе стороны на [latex] {r} _ {3}, [/ latex], получаем

[латекс] \ begin {array} {cc} \ hfill {m} _ {3} & = \ frac {{r} _ {1}} {{r} _ {3}} \, {m} _ { 1} + \ frac {{r} _ {2}} {{r} _ {3}} \, {m} _ {2} + \ frac {r} {{r} _ {3}} \, m \ hfill \\ & = \ frac {70} {30} \, (50.{2}} = 5.8 \, \ text {N}. \ Hfill \ end {array} [/ latex]

Значение

Обратите внимание, что (рисунок) не зависит от значения г . Таким образом, баланс крутящего момента может использоваться для измерения массы, поскольку изменения значений g на поверхности Земли не влияют на эти измерения. Это не относится к пружинным весам, поскольку они измеряют силу.

Проверьте свое понимание

Повторите (рисунок), используя левый конец измерительной ручки для расчета крутящего момента; то есть, поместив ось на левый конец измерительной ручки.

В следующем примере мы покажем, как использовать первое условие равновесия (уравнение для сил) в векторной форме, заданной (Рисунок) и (Рисунок). Мы представляем это решение, чтобы проиллюстрировать важность подходящего выбора системы отсчета. Хотя все инерциальные системы отсчета эквивалентны, а численные решения, полученные в одном кадре, такие же, как и в любом другом, неподходящий выбор системы отсчета может сделать решение довольно длинным и запутанным, тогда как мудрый выбор системы отсчета делает решение простым.Мы покажем это в эквивалентном решении той же проблемы. Этот конкретный пример иллюстрирует применение статического равновесия к биомеханике.

Пример

Силы в предплечье

Тяжелоатлет держит в предплечье гирю весом 50,0 фунтов (эквивалент 222,4 Н), как показано на (Рисунок). Его предплечье расположено на [latex] \ beta = 60 \ text {°} [/ latex] относительно его плеча. Предплечье поддерживается сокращением двуглавой мышцы, которое вызывает вращающий момент вокруг локтя.Предполагая, что напряжение в двуглавой мышце действует в вертикальном направлении, определяемом силой тяжести, какое напряжение должна прикладывать мышца, чтобы удерживать предплечье в показанном положении? Какая сила действует на локтевой сустав? Предположим, что вес предплечья незначителен. Дайте окончательные ответы в единицах СИ.

Рисунок 12.11 Предплечье вращается вокруг локтя (E) за счет сокращения мышцы двуглавой мышцы, что вызывает напряжение [латекс] {\ overset {\ to} {T}} _ {\ text {M}}. [/ латекс]

Стратегия

Мы идентифицируем три силы, действующие на предплечье: неизвестная сила [латекс] \ overset {\ to} {F} [/ latex] в локте; неизвестное напряжение [латекс] {\ overset {\ to} {T}} _ {\ text {M}} [/ latex] в мышце; и вес [латекс] \ overset {\ to} {w} [/ latex] с величиной [латекс] w = 50 \, \ text {lb}.[/ latex] Мы принимаем систему отсчета с осью x вдоль предплечья и шарниром в локте. Вертикальное направление — это направление веса, которое совпадает с направлением плеча. Ось x составляет угол [latex] \ beta = 60 \ text {°} [/ latex] с вертикалью. Ось y перпендикулярна оси x . Теперь создадим диаграмму свободного тела для предплечья. Сначала мы рисуем оси, точку поворота и три вектора, представляющие три идентифицированные силы.Затем мы определяем угол [latex] \ beta [/ latex] и представляем каждую силу ее компонентами x и y , не забывая перечеркнуть исходный вектор силы, чтобы избежать двойного счета. Наконец, мы помечаем силы и их рычаги. Схема свободного тела для предплечья представлена ​​на (Рисунок). На этом этапе мы готовы создать условия равновесия для предплечья. Каждая сила имеет компоненты x и y ; следовательно, у нас есть два уравнения для первого условия равновесия, по одному уравнению для каждой составляющей чистой силы, действующей на предплечье.

Рисунок 12.12 Схема свободного тела для предплечья: шарнир расположен в точке E (локоть).

Обратите внимание, что в нашей системе отсчета вклад во второе условие равновесия (для крутящих моментов) происходит только от x -компонент сил, потому что все x -компоненты сил параллельны плечам их рычагов, поэтому что для любого из них у нас есть [latex] \ text {sin} \, \ theta = 0 [/ latex] на (Рисунок). Для компонентов y мы имеем [latex] \ theta = ± 90 \ text {°} [/ latex] на (Рисунок).Также обратите внимание, что крутящий момент силы в локте равен нулю, потому что эта сила приложена к шарниру. Таким образом, вклад в чистый крутящий момент вносится только крутящими моментами [латекса] {T} _ {y} [/ latex] и [латекса] {w} _ {y}. [/ латекс]

Решение

Из диаграммы свободного тела видно, что составляющая x чистой силы удовлетворяет уравнению

[латекс] + {F} _ {x} + {T} _ {x} — {w} _ {x} = 0 [/ латекс]

и y -компонент чистой силы удовлетворяет

[латекс] + {F} _ {y} + {T} _ {y} — {w} _ {y} = 0.[/ латекс]

(рисунок) и (рисунок) — это два уравнения первого условия равновесия (для сил). Затем мы читаем из диаграммы свободного тела, что чистый крутящий момент вдоль оси вращения равен

.

[латекс] + {r} _ {T} {T} _ {y} — {r} _ {w} {w} _ {y} = 0. [/ латекс]

(рисунок) — второе условие равновесия (по крутящим моментам) для предплечья. На диаграмме свободного тела показано, что плечи рычага представляют собой [латекс] {r} _ {T} = 1.5 \, \ text {in} \ text {.} [/ Latex] и [латекс] {r} _ {w} = 13,0 \, \ текст {в} \ текст {.} [/ latex] На этом этапе нам не нужно преобразовывать дюймы в единицы СИ, потому что, пока эти единицы согласованы на (рис.), они взаимно сокращаются. Снова используя диаграмму свободного тела, находим величины составляющих сил:

[латекс] \ begin {array} {ccc} \ hfill {F} _ {x} & = \ hfill & F \, \ text {cos} \, \ beta = F \, \ text {cos} \, 60 \ text {°} = F \, \ text {/} \, 2 \ hfill \\ \ hfill {T} _ {x} & = \ hfill & T \, \ text {cos} \, \ beta = T \ , \ text {cos} \, 60 \ text {°} = T \, \ text {/} \, 2 \ hfill \\ \ hfill {w} _ {x} & = \ hfill & w \, \ text { cos} \, \ beta = w \, \ text {cos} \, 60 \ text {°} = w \, \ text {/} \, 2 \ hfill \\ \ hfill {F} _ {y} & = \ hfill & F \, \ text {sin} \, \ beta = F \, \ text {sin} \, 60 \ text {°} = F \ sqrt {3} \, \ text {/} \, 2 \ hfill \\ \ hfill {T} _ {y} & = \ hfill & T \, \ text {sin} \, \ beta = T \, \ text {sin} \, 60 \ text {°} = T \ sqrt {3} \, \ text {/} \, 2 \ hfill \\ \ hfill {w} _ {y} & = \ hfill & w \, \ text {sin} \, \ beta = w \, \ text { sin} \, 60 \ text {°} = w \ sqrt {3} \, \ text {/} \, 2.\ hfill \ end {array} [/ latex]

Мы подставляем эти величины в (Рисунок), (Рисунок) и (Рисунок), чтобы получить, соответственно,

[латекс] \ begin {array} {ccc} \ hfill F \, \ text {/} \, 2 + T \, \ text {/} \, 2-w \, \ text {/} \, 2 & = \ hfill & 0 \ hfill \\ \\ \ hfill F \ sqrt {3} \, \ text {/} \, 2 + T \ sqrt {3} \, \ text {/} \, 2-w \ sqrt { 3} \, \ text {/} \, 2 & = \ hfill & 0 \ hfill \\ \\ \ hfill {r} _ {T} T \ sqrt {3} \, \ text {/} \, 2- { r} _ {w} w \ sqrt {3} \, \ text {/} \, 2 & = \ hfill & 0. \ hfill \ end {array} [/ latex]

Когда мы упрощаем эти уравнения, мы видим, что остались только два независимых уравнения для двух неизвестных величин силы, F и T , потому что (Рисунок) для компонента x эквивалентен (Рисунок) для компонента y .Таким образом, мы получаем первое условие равновесия для сил

[латекс] Ж + Т-ш = 0 [/ латекс]

и второе условие равновесия моментов

[латекс] {r} _ {T} T- {r} _ {w} w = 0. [/ латекс]

Величина напряжения в мышце получается путем решения (Рисунок):

[латекс] T = \ frac {{r} _ {w}} {{r} _ {T}} \, w = \ frac {13.0} {1.5} \, \ text {(50 фунтов)} = 433 \, \ frac {1} {3} \ text {lb} \ simeq 433.3 \, \ text {lb.} [/ latex]

Сила в локте определяется решением (рисунок):

[латекс] F = w-T = 50.0 \, \ text {lb} -433.3 \, \ text {lb} = — 383.3 \, \ text {lb.} [/ Latex]

Отрицательный знак в уравнении говорит нам, что действительная сила в локте антипараллельна рабочему направлению, принятому для построения диаграммы свободного тела. В окончательном ответе мы конвертируем силы в единицы силы СИ. Ответ

[латекс] \ begin {array} {c} F = 383,3 \, \ text {lb} = 383,3 (4,448 \, \ text {N}) = 1705 \, \ text {N вниз} \\ T = 433,3 \ , \ text {lb} = 433.3 (4.448 \, \ text {N}) = 1927 \, \ text {N вверх.} \ end {array} [/ latex]

Значение

Здесь стоит отметить два важных момента.Первый касается преобразования в единицы СИ, который может быть выполнен в самом конце решения, если мы сохраняем согласованность в единицах. Второй важный вопрос касается шарнирных соединений, например, локтевого. При первоначальном анализе проблемы следует всегда предполагать, что шарнирные соединения прилагают силу в произвольном направлении , а затем вы должны решить для всех компонентов шарнирной силы независимо. В этом примере сила в локтевом суставе оказывается вертикальной, потому что задача предполагает, что напряжение бицепса также является вертикальным.Однако такое упрощение не является общим правилом.

Решение

Предположим, мы принимаем опорную систему с направлением оси y вдоль 50-фунтового груза и осью, расположенной в колене. В этой системе отсчета все три силы имеют только y -компонент, поэтому у нас есть только одно уравнение для первого условия равновесия (для сил). Мы рисуем диаграмму свободного тела для предплечья, как показано на (Рисунок), с указанием оси поворота, действующих сил и их плеч рычагов по отношению к оси поворота, а также углов [латекс] {\ theta} _ {T} [/ латекс] и [латекс] {\ theta} _ {w} [/ latex], которые заставляют [латекс] {\ overset {\ to} {T}} _ {\ text {M}} [/ latex] и [латекс ] \ overset {\ to} {w} [/ latex] (соответственно) сделать своими рычагами.В определении крутящего момента, приведенном на (Рисунок), угол [латекс] {\ theta} _ {T} [/ latex] — это угол направления вектора [латекс] {\ overset {\ to} {T}} _ {\ text {M}}, [/ latex] отсчитывается от против часовой стрелки до от радиального направления плеча рычага, который всегда указывает от оси. По тому же соглашению угол [латекс] {\ theta} _ {w} [/ latex] измеряется против часовой стрелки от радиального направления плеча рычага к вектору [латекс] \ overset {\ to} {w} . [/ latex] Выполненный таким образом ненулевой крутящий момент легче всего вычислить путем прямой подстановки в (рисунок) следующим образом:

[латекс] \ begin {array} {cc} {\ tau} _ {T} = {r} _ {T} T \, \ text {sin} \, {\ theta} _ {T} = {r} _ {T} T \, \ text {sin} \, \ beta = {r} _ {T} T \, \ text {sin} \, 60 \ text {°} = + {r} _ {T} T \ sqrt {3} \, \ text {/} \, 2 \\ {\ tau} _ {w} = {r} _ {w} w \, \ text {sin} \, {\ theta} _ {w } = {r} _ {w} w \, \ text {sin} (\ beta +180 \ text {°}) = \ text {-} {r} _ {w} w \, \ text {sin} \ , \ beta = \ text {-} {r} _ {w} w \ sqrt {3} \, \ text {/} \, 2.\ end {array} [/ latex]

Рис. 12.13 Диаграмма свободного тела для предплечья для эквивалентного решения. Ось находится в точке Е (колено).

Второе условие равновесия, [латекс] {\ tau} _ {T} + {\ tau} _ {w} = 0, [/ latex] теперь может быть записано как

[латекс] {r} _ {T} T \ sqrt {3} \, \ text {/} \, 2- {r} _ {w} w \ sqrt {3} \, \ text {/} \, 2 = 0. [/ латекс]

Из диаграммы свободного тела первое условие равновесия (для сил) равно

.

[латекс] \ text {-} F + T-w = 0.[/ латекс]

(рисунок) идентичен (рисунок) и дает результат [латекс] T = 433,3 \, \ text {lb}. [/ latex] (рисунок) дает

[латекс] F = T-w = 433,3 \, \ text {lb} -50,0 \, \ text {lb} = 383,3 \, \ text {lb.} [/ Latex]

Мы видим, что эти ответы идентичны нашим предыдущим ответам, но второй выбор системы отсчета приводит к эквивалентному решению, которое является более простым и быстрым, поскольку не требует разделения сил на их прямоугольные составляющие.

Проверьте свое понимание

Повторите (рисунок), предполагая, что предплечье представляет собой объект одинаковой плотности, который весит 8.896 Н.

Показать решение

[латекс] T = \ text {1963 N}; \, \ text {F} = 1732 \, \ text {N} [/ latex]

Пример

Лестница, упирающаяся в стену

Унифицированная лестница имеет длину [латекс] L = 5,0 \, \ text {m} [/ latex] и весит 400,0 Н. Лестница упирается в скользкую вертикальную стену, как показано на (Рисунок). Угол наклона между лестницей и черновым полом составляет [латекс] \ beta = 53 \ text {°}. [/ latex] Найдите силы реакции пола и стены на лестнице и коэффициент статического трения [латекс] {\ mu} _ {\ text {s}} [/ latex] на стыке лестницы с пол, предотвращающий скольжение лестницы.

Рисунок 12.14 Лестница длиной 5,0 м упирается в стену без трения.

Стратегия

Мы можем выделить четыре силы, действующие на лестницу. Первая сила — это сила нормальной реакции Н от пола в вертикальном направлении вверх. Вторая сила — это сила статического трения [латекс] f = {\ mu} _ {\ text {s}} N [/ latex], направленная горизонтально по полу к стене — эта сила предотвращает скольжение лестницы. Эти две силы действуют на лестницу в точке ее контакта с полом.Третья сила — это вес против лестницы, прикрепленный к ее КМ, расположенной посередине между ее концами. Четвертая сила — это нормальная сила реакции F от стены в горизонтальном направлении от стены, приложенная в точке контакта со стеной. Других сил нет, потому что стена скользкая, что означает отсутствие трения между стеной и лестницей. На основе этого анализа мы принимаем систему отсчета с осью y в вертикальном направлении (параллельно стене) и осью x в горизонтальном направлении (параллельно полу).В этом кадре каждая сила имеет либо горизонтальную, либо вертикальную составляющую, но не обе, что упрощает решение. Подбираем ось в точке соприкосновения с полом. На диаграмме свободного тела для лестницы мы указываем ось, все четыре силы и их плечи рычагов, а также углы между плечами рычагов и силами, как показано на (Рисунок). При нашем выборе положения оси вращения отсутствует крутящий момент ни от нормальной силы реакции N , ни от статического трения f , потому что они оба действуют на ось.

Рисунок 12.15 Схема свободного тела для лестницы, упирающейся в стену без трения.

Решение

На диаграмме свободного тела чистая сила в направлении x равна

.

[латекс] + f-F = 0 [/ латекс]

чистая сила в направлении y составляет

[латекс] + N = 0 [/ латекс]

, а чистый крутящий момент по оси вращения в точке поворота равен

.

[латекс] {\ tau} _ {w} + {\ tau} _ {F} = 0. [/ латекс]

, где [latex] {\ tau} _ {w} [/ latex] — это крутящий момент веса w , а [latex] {\ tau} _ {F} [/ latex] — крутящий момент реакции F .Из диаграммы свободного тела мы определяем, что плечо рычага реакции у стены — это [латекс] {r} _ {F} = L = 5,0 \, \ text {m} [/ латекс], а плечо рычага вес [латекс] {r} _ {w} = L \, \ text {/} \, 2 = 2.5 \, \ text {m}. [/ latex] С помощью диаграммы свободного тела мы определяем углы, которые будут использоваться на (Рисунок) для крутящих моментов: [latex] {\ theta} _ {F} = 180 \ text {°} — \ beta [ / latex] для крутящего момента от силы реакции со стеной и [latex] {\ theta} _ {w} = 180 \ text {°} + (90 \ text {°} — \ beta) [/ latex] для крутящий момент из-за веса.Теперь мы готовы использовать (рисунок) для вычисления крутящих моментов:

[латекс] \ begin {array} {cc} {\ tau} _ {w} = {r} _ {w} w \, \ text {sin} \, {\ theta} _ {w} = {r} _ {w} w \, \ text {sin} (180 \ text {°} +90 \ text {°} — \ beta) = — \ frac {L} {2} w \, \ text {sin} (90 \ text {°} — \ beta) = — \ frac {L} {2} w \, \ text {cos} \, \ beta \\ {\ tau} _ {F} = {r} _ {F} F \, \ text {sin} \, {\ theta} _ {F} = {r} _ {F} F \, \ text {sin} (180 \ text {°} — \ beta) = LF \, \ text {sin} \, \ beta. \ end {array} [/ latex]

Подставляем значения крутящего момента в (рисунок) и решаем для [латекс] F: [/ латекс]

[латекс] \ begin {array} {} \\ \ hfill — \ frac {L} {2} w \, \ text {cos} \, \ beta + LF \, \ text {sin} \, \ beta & = \ hfill & 0 \ hfill \\ \ hfill F = \ frac {w} {2} \, \ text {cot} \, \ beta = \ frac {400.0 \, \ text {N}} {2} \, \ text {cot} \, 53 \ text {°} & = \ hfill & 150.7 \, \ text {N} \ hfill \ end {array} [/ latex ]

Мы получаем нормальную силу реакции с полом, решая (рисунок): [латекс] N = w = 400.0 \, \ text {N}. [/ latex] Величина трения получается путем решения (рисунок): [latex] f = F = 150,7 \, \ text {N}. [/ latex] Коэффициент статического трения [латекс] {\ mu} _ {\ text {s}} = f \, \ text {/} \, N = 150,7 \, \ text {/} \, 400,0 = 0,377. [/ латекс]

Чистая сила, действующая на лестницу в точке контакта с полом, представляет собой векторную сумму нормальной реакции пола и сил статического трения:

[латекс] {\ overset {\ to} {F}} _ {\ text {floor}} = \ overset {\ to} {f} + \ overset {\ to} {N} = \ text {(150.{-1} (400.0 \, \, \ text {/} \, 150.7) = 69.3 \ text {°} \, \ text {над полом.} [/ Latex]

Здесь мы должны выделить два общих замечания о практическом использовании. Во-первых, обратите внимание, что когда мы выбираем точку поворота, нет никаких ожиданий, что система действительно развернется вокруг выбранной точки. Лестница в этом примере совсем не вращается, а твердо стоит на полу; тем не менее, его точка контакта с полом — хороший выбор для шарнира. Во-вторых, обратите внимание, когда мы используем (рисунок) для расчета отдельных крутящих моментов, нам не нужно разделять силы на их нормальные и параллельные компоненты по отношению к направлению плеча рычага, и нам не нужно учитывать смысл крутящий момент.Если угол на (Рис.) Правильно определен — с помощью диаграммы свободного тела — как угол, измеренный против часовой стрелки от направления плеча рычага к направлению вектора силы, (Рис.) Дает как величину и чувство крутящего момента. Это связано с тем, что крутящий момент представляет собой векторное произведение вектора рычага на плечо, пересекаемого с вектором силы, и (рисунок) выражает прямоугольную составляющую этого векторного произведения вдоль оси вращения.

Значение

Этот результат не зависит от длины лестницы, поскольку L отменяется во втором состоянии равновесия (рисунок).Независимо от длины или длины лестницы, если ее вес составляет 400 Н, а угол с полом составляет [латекс] 53 \ text {°}, [/ latex], наши результаты остаются в силе. Но лестница соскользнет, ​​если чистый крутящий момент станет отрицательным (рисунок). Это происходит для некоторых углов, когда коэффициент статического трения недостаточен для предотвращения скольжения лестницы.

Проверьте свое понимание

Для ситуации, описанной на (Рисунок), определите значения коэффициента [латекс] {\ mu} _ {\ text {s}} [/ latex] статического трения, при котором лестница начинает скользить, учитывая, что [латекс] \ beta [/ latex] — угол между лестницей и полом.

Показать решение

[латекс] {\ mu} _ {s} <0,5 \, \ text {cot} \, \ beta [/ latex]

Пример

Усилие на дверных петлях

Распашная дверь, которая весит [латекс] w = 400,0 \, \ text {N} [/ latex], поддерживается петлями A и B , так что дверь может вращаться вокруг вертикальной оси, проходящей через петли (рис. ). Дверь имеет ширину [латекс] b = 1,00 \, \ text {m}, [/ latex], а дверная плита имеет однородную массовую плотность. Петли располагаются симметрично у края двери таким образом, чтобы вес двери равномерно распределялся между ними.Петли разделены расстоянием [латекс] a = 2,00 \, \ text {m}. [/ latex] Найдите силы на петлях, когда дверь приоткрыта.

Рисунок 12.16 Распашная вертикальная дверь 400-N поддерживается двумя петлями, прикрепленными в точках A и B.

Стратегия

Силы, которые дверь оказывает на петли, можно найти, просто изменив направление сил, которые петли воздействуют на дверь. Следовательно, наша задача найти силы от петель на двери.На дверную плиту действуют три силы: неизвестная сила [латекс] \ overset {\ to} {A} [/ latex] от петли [латекс] A, [/ латекс] неизвестная сила [латекс] \ overset {\ to} {B} [/ latex] из петли [латекс] B, [/ latex] и известного веса [латекс] \ overset {\ to} {w} [/ latex], прикрепленного в центре масс дверной плиты. CM расположен в геометрическом центре двери, потому что плита имеет однородную массовую плотность. Мы принимаем прямоугольную систему отсчета с осью y вдоль направления силы тяжести и осью x в плоскости плиты, как показано на панели (a) (Рисунок), и разрешаем все силы на их прямоугольные составляющие.Таким образом, у нас есть четыре неизвестных составляющих силы: две составляющие силы [латекс] \ overset {\ to} {A} [/ latex] [latex] ({A} _ {x} [/ latex] и [latex] {A} _ {y}), [/ latex] и два компонента силы [latex] \ overset {\ to} {B} [/ latex] [latex] ({B} _ {x} [/ latex] и [латекс] {B} _ {y}). [/ latex] На схеме свободного тела мы представляем две силы на шарнирах их векторными компонентами, предполагаемые ориентации которых произвольны. Поскольку есть четыре неизвестных [латекс] ({A} _ {x}, [/ latex] [latex] {B} _ {x}, [/ latex] [latex] {A} _ {y}, [/ latex ] и [latex] {B} _ {y}), [/ latex] мы должны составить четыре независимых уравнения.Одно уравнение — это условие равновесия сил в направлении x . Второе уравнение — это условие равновесия сил в направлении y . Третье уравнение — это условие равновесия крутящих моментов при вращении вокруг шарнира. Поскольку вес равномерно распределяется между петлями, мы получаем четвертое уравнение [латекс] {A} _ {y} = {B} _ {y}. [/ latex] Чтобы установить условия равновесия, мы рисуем диаграмму свободного тела и выбираем точку поворота на верхнем шарнире, как показано на панели (b) (Рисунок).Наконец, мы решаем уравнения для неизвестных компонентов силы и находим силы.

Рисунок 12.17 (a) Геометрия и (b) диаграмма свободного тела двери.

Решение

Из диаграммы свободного тела для двери мы имеем первое условие равновесия сил:

[латекс] \ begin {array} {} \\ \ text {in} \, x \ text {-direction:} \ hfill & \, — {A} _ {x} + {B} _ {x} = 0 \ enspace⇒ \ enspace {A} _ {x} = {B} _ {x} \ hfill \\ \ text {in} \, y \ text {-direction:} \ hfill & + {A} _ {y } + {B} _ {y} -w = 0 \ enspace⇒ \ enspace {A} _ {y} = {B} _ {y} = \ frac {w} {2} = \ frac {400.0 \, \ text {N}} {2} = 200.0 \, \ text {N.} \ Hfill \ end {array} [/ latex]

Выбираем шарнир в точке P (верхний шарнир, согласно диаграмме свободного тела) и записываем второе условие равновесия для крутящих моментов при вращении вокруг точки P :

[латекс] \ text {pivot at} \, P \ text {:} \, {\ tau} _ {w} + {\ tau} _ {Bx} + {\ tau} _ {By} = 0. [/ латекс]

Мы используем диаграмму свободного тела, чтобы найти все члены в этом уравнении:

[латекс] \ begin {array} {ccc} \ hfill {\ tau} _ {w} & = \ hfill & dw \, \ text {sin} (\ text {-} \ beta) = \ text {-} dw \, \ text {sin} \, \ beta = \ text {-} dw \ frac {b \, \ text {/} \, 2} {d} = \ text {-} w \ frac {b} { 2} \ hfill \\ \ hfill {\ tau} _ {Bx} & = \ hfill & a {B} _ {x} \ text {sin} \, 90 \ text {°} = + a {B} _ { x} \ hfill \\ \ hfill {\ tau} _ {By} & = \ hfill & a {B} _ {y} \ text {sin} \, 180 \ text {°} = 0.\ hfill \ end {array} [/ latex]

При оценке [latex] \ text {sin} \, \ beta, [/ latex] мы используем геометрию треугольника, показанную в части (a) рисунка. Теперь подставляем эти моменты в (рисунок) и вычисляем [латекс] {B} _ {x}: [/ latex]

[латекс] \ text {pivot at} \, P \ text {:} \, \ text {-} w \, \ frac {b} {2} + a {B} _ {x} = 0 \ enspace⇒ \ enspace {B} _ {x} = w \, \ frac {b} {2a} = (400.0 \, \ text {N}) \, \ frac {1} {2 · 2} = 100.0 \, \ text {N.} [/ латекс]

Следовательно, величины горизонтальных составляющих сил равны [латекс] {A} _ {x} = {B} _ {x} = 100.0 \, \ text {N}. [/ latex] Силы на двери

[латекс] \ begin {array} {} \\ \ text {на верхнем шарнире:} \, {\ overset {\ to} {F}} _ {A \, \ text {on door}} = — 100,0 \, \ text {N} \ hat {i} +200.0 \, \ text {N} \ hat {j} \ hfill \\ \ text {на нижнем шарнире:} {\ overset {\ to} {F}} _ {B \, \ text {на двери}} = \ text {+} 100.0 \, \ text {N} \ hat {i} +200.0 \, \ text {N} \ hat {j}. \ Hfill \ end {array} [/ latex]

Силы на петлях находятся из третьего закона Ньютона как

[латекс] \ begin {array} {cc} \ text {на верхнем шарнире:} \, {\ overset {\ to} {F}} _ {\ text {door on} \, A} = 100.0 \, \ text {N} \ hat {i} -200.0 \, \ text {N} \ hat {j} \ hfill \\ \ text {на нижнем шарнире:} \, {\ overset {\ to} { F}} _ {\ text {door on} \, B} = — 100.0 \, \ text {N} \ hat {i} -200.0 \, \ text {N} \ hat {j}. \ Hfill \ end { array} [/ latex]

Значение

Обратите внимание, что если бы задача была сформулирована без предположения о том, что вес равномерно распределен между двумя петлями, мы не смогли бы ее решить, потому что количество неизвестных было бы больше, чем количество уравнений, выражающих условия равновесия.

Проверьте свое понимание

Решите проблему, показанную на (Рисунок), приняв положение поворота в центре масс.

Показать решение

[латекс] {\ overset {\ to} {F}} _ {\ text {door on} \, A} = 100.0 \, \ text {N} \ hat {i} -200.0 \, \ text {N} \ hat {j} \, \ text {;} \, {\ overset {\ to} {F}} _ {\ text {door on} \, B} = — 100.0 \, \ text {N} \ hat { i} -200.0 \, \ text {N} \ hat {j} [/ latex]

Проверьте свое понимание

Человек массой 50 кг стоит на расстоянии 1,5 м от одного конца унифицированных лесов длиной 6,0 м и массой 70.0 кг. Найдите натяжение двух вертикальных тросов, поддерживающих подмости.

Показать решение

711,0 Н; 466.0 с.

Проверьте свое понимание

Знак 400.0-N висит на конце форменной стойки. Стойка имеет длину 4,0 м и вес 600,0 Н. Стойка поддерживается петлей на стене и тросом, другой конец которого привязан к стене на высоте 3,0 м над левым концом стойки. Найдите натяжение опорного троса и усилие петли на стойке.

Показать решение

1167 Н; 980 Н направлено вверх на [латекс] 18 \ text {°} [/ latex] над горизонтом

Сводка

  • Разнообразные инженерные задачи могут быть решены путем применения условий равновесия для твердых тел.
  • В приложениях идентифицируйте все силы, которые действуют на твердое тело, и отметьте их рычаги, вращающиеся вокруг выбранной оси вращения. Постройте диаграмму свободного тела для тела. Чистые внешние силы и крутящие моменты можно четко определить по правильно построенной диаграмме свободного тела.Таким образом, вы можете установить первое условие равновесия для сил и второе условие равновесия для крутящих моментов.
  • При установке условий равновесия мы можем принять любую инерциальную систему отсчета и любое положение точки поворота. Все варианты приводят к одному ответу. Однако некоторые варианты могут чрезмерно усложнить процесс поиска решения. Независимо от того, какой выбор мы делаем, мы получаем один и тот же ответ. Единственный способ овладеть этим навыком — это практика.

Концептуальные вопросы

Можно ли упереть лестницу в неровную стену, когда пол без трения?

Покажите, как с помощью пружинных весов и простой точки опоры можно взвесить объект, вес которого превышает максимальное показание на весах.

Художник поднимается по лестнице. Будет ли лестница соскользнуть с большей вероятностью, когда художник находится внизу или вверху?

Проблемы

Равномерная доска стоит на ровной поверхности, как показано ниже. Доска имеет массу 30 кг и длину 6,0 м. Какую массу можно поместить на его правый конец, прежде чем он наклонится? ( Подсказка: Когда доска собирается опрокинуться, она соприкасается с поверхностью только по краю, который становится мгновенной осью вращения.)

Унифицированные качели, показанные ниже, уравновешены на точке опоры, расположенной 3.0 м от левого конца. Маленький мальчик справа имеет массу 40 кг, а больший мальчик слева имеет массу 80 кг. Какая масса у доски?

Чтобы вытащить свою машину из грязи, мужчина привязывает один конец веревки к переднему бамперу, а другой конец — к дереву на расстоянии 15 м, как показано ниже. Затем он тянет за центр веревки с силой 400 Н, в результате чего ее центр смещается на 0,30 м, как показано. Какова сила троса на машине?

Униформа 40.Подмости 0 кг длиной 6,0 м поддерживаются двумя световыми кабелями, как показано ниже. Маляр весом 80,0 кг стоит на расстоянии 1,0 м от левого конца строительных лесов, а его малярное оборудование — в 1,5 м от правого конца. Если натяжение левого троса вдвое больше, чем правого троса, найдите натяжение тросов и массу оборудования.

Покажи ответ

кабель правый, 444,3 Н; левый кабель, 888,5 Н; вес оборудования 156,8 Н; 16,0 кг

Когда структура, показанная ниже, поддерживается в точке P , она находится в состоянии равновесия.Найдите величину силы F и силу, приложенную в точке P . Вес конструкции незначительный.

Чтобы подняться на крышу, человек (массой 70,0 кг) ставит алюминиевую лестницу длиной 6,00 м (массой 10,0 кг) на бетонную площадку с основанием лестницы на расстоянии 2,00 м от дома. Лестница упирается в пластиковый водосточный желоб, который, как мы можем предположить, не имеет трения. Центр масс лестницы находится на расстоянии 2,00 м от низа. Человек стоит 3.00 м от дна. Найдите нормальные силы реакции и трения лестницы у ее основания.

Единая горизонтальная стойка весит 400,0 Н. Один конец стойки прикреплен к шарнирной опоре у стены, а другой конец стойки прикреплен к знаку весом 200,0 Н. Стойка также поддерживается прикрепленным тросом. между концом подкоса и стеной. Предполагая, что весь вес знака прикреплен к самому концу стойки, найдите натяжение троса и усилие на шарнире стойки.

Предплечье, показанное ниже, расположено под углом [латекс] \ тета [/ латекс] по отношению к плечу, а в руке удерживается груз весом 5,0 кг. Общая масса предплечья и кисти составляет 3,0 кг, а их центр масс находится на расстоянии 15,0 см от локтя. (a) Какова величина силы, которую мышца двуглавой мышцы оказывает на предплечье для [latex] \ theta = 60 \ text {°} \ text {?} [/ latex] (b) Какова величина силы на локтевой сустав на такой же угол? (c) Как эти силы зависят от угла [латекс] \ theta? [/ латекс]

Показать решение

а.539 Н; б. 461 Н; c. не зависят от угла

Унифицированная стрела, показанная ниже, весит 3000 Н. Она поддерживается горизонтальной растяжкой и шарнирной опорой в точке A . Какие силы действуют на стрелу из-за троса и опоры на A ? Действует ли сила A вдоль стрелы?

Унифицированная стрела, показанная ниже, весит 700 Н, а объект, свисающий с ее правого конца, весит 400 Н. Стрела поддерживается световым кабелем и шарниром на стене.Рассчитайте натяжение троса и усилие на шарнире на стреле. Действует ли сила на шарнире вдоль стрелы?

Показать решение

напряжение 778 Н; на петле 778 N на [латексе] 45 \ text {°} [/ latex] выше горизонтали; нет

Стрела 12,0 м, AB , крана, поднимающего груз массой 3000 кг, показана ниже. Центр масс стрелы находится в ее геометрическом центре, а масса стрелы составляет 1000 кг. Для показанного положения рассчитайте натяжение троса T и усилие на оси A .

Унифицированный люк, показанный ниже, имеет размер 1,0 м на 1,5 м и весит 300 Н. Он поддерживается одной петлей (H) и легкой веревкой, привязанной между серединой двери и полом. Дверь удерживается в показанном положении, где ее плита составляет [латекс] 30 \ text {°} [/ latex] угол с горизонтальным полом, а веревка составляет [латекс] 20 \ text {°} [/ latex] угол с полом. Найдите натяжение веревки и усилие на петле.

Показать решение

1500 Н; 1620 Н при [латекс] 30 \ text {°} [/ latex]

Мужчина весом 90 кг ходит на козле, как показано ниже.Длина козлы 2,0 м, высота 1,0 м, масса 25,0 кг. Рассчитайте нормальную силу реакции на каждую ногу в точке контакта с полом, когда человек находится на расстоянии 0,5 м от дальнего конца козлы. ( Подсказка: На каждом конце сначала найдите общую силу реакции. Эта сила реакции представляет собой векторную сумму двух сил реакции, каждая из которых действует вдоль одной ноги. Нормальная сила реакции в точке контакта с полом является нормальной (с относительно пола) составляющая этой силы.)

Внутренние силы

Внутренние силы:

Определения:

Для расчета конструктивного элемента требуется исследование нагрузки, действующей внутри элемента, чтобы убедиться, что материал может сопротивляться переносу груза.

А также расчет сил и моменты, передаваемые через соединения между элементами, мы также можем рассчитать внутренние силы, которые одна часть члена оказывает на другую. Внутренние нагрузки можно найти методом разделов .

Чтобы рассчитать эти внутренние силы, просто:

  1. Нарисуйте диаграмма свободного тела всего тела,
  2. Найти реакции на внешних опорах,
  3. Найти реакции на соединениях,
  4. Оставить все грузы в их точном местонахождении,
  5. Пройти разрез элемента перпендикулярно его оси в точке, где внутренний нагрузки подлежат уточнению,
  6. Draw FBD of отрезанный член,
  7. для 2D Задача найти N, V, M (нормальная сила, поперечная сила и момент соответственно) из положения равновесия.

Например, рассмотрим следующее участник:

Если вам интересно узнать силы и моменты, которые передаются через элемент в точке C ; ты можешь пройти воображаемый разрез, разрезанный через C, разрезающий балку на два сегмента:

Примечание: силы и момент, приложенные левой стороной к правой, равны величина, но противоположная по направлению силам, прикладываемым правой стороной к левая сторона.Внутренние нагрузки на всю балку проявляются как внешние нагрузки на FBD. разделы.

Каждый сегмент должен быть в равновесии. Применяем уравнения равновесия сегменту и решите для компонентов внутренней силы.

Балка двумерная

Балка трехмерная

Пример:

Пример 1

Пример 1: Для показанной рамы и нагрузки рассчитайте реакции на опорах A и E.

Схема со свободным телом: Конструкция закреплена в точке A и закреплена в точке E. Кроме того, элемент BDE опирается на балку ABC через ролик в точке B. Нарисуем диаграмму конструкции со свободным телом следующим образом. целая первая.

Обратите внимание, что мы имеем дело с двумерной системой сил, что означает, что мы можем использовать только три уравнения равновесия. Когда мы смотрим на диаграмму свободного тела выше, мы видим пять неизвестных реакций. так что нам делать?

Если бы член BDE был неотъемлемой частью структуры, мы не смогли бы решить реакции, используя только уравнения равновесия.Однако в этом случае в точке B. Из нашего предыдущего обсуждения опорных реакций мы знаем, что в таком случае существует только одна неизвестная сила в нормальном направлении. Признание этого факта само по себе является ключом к решению этой проблемы.

Поскольку мы знаем, что если структура находится в состоянии равновесия, каждая ее часть также находится в равновесии, мы можем разделить структуру на два отдельных элемента, как показано ниже.

Мы не можем начать анализ, используя диаграмму свободного тела балки ABC, поскольку она содержит 4 неизвестных нагрузки.Итак, мы начнем с члена BDE, поскольку у него есть только три неизвестные силы. Более пристальный взгляд на члена BDE показывает, что на самом деле он состоит из трех частей. Откуда мы это знаем? Итак, BDE находится под действием одной силы в точке B, одной силы в точке D и одной силы в точке E, которые являются равнодействующими REx и REy. Чтобы обеспечить равновесие сил и моментов, эти три силы должны действовать одновременно. В этом случае точкой параллелизма является точка D. Это означает, что y составляющая силы в точке E (т.е. REy) фактически равна нулю.Хотя это признание помогает прийти к окончательному ответу. эффективно, это несущественно при анализе неизвестных сил. Это связано с тем, что диаграмма свободного тела BDE включает только три неизвестных силы, поэтому мы может решить их, используя три уравнения равновесия.

Давайте начнем решение, используя равновесие моментов с точкой E как центром момента и направлением против часовой стрелки как положительным. Мы также разделили силу 50 Н на две составляющие по осям x, y.

Положительный знак, подтверждающий правильность принятого направления. Теперь, суммируя силы в направлениях x и y, мы решаем неизвестные реакции в точке E.

Отрицательный знак указывает, что направление противоположно тому, которое показано на диаграмме свободного тела. Следовательно, REx указывает налево, а не направо.

Проверка результатов: Напомним, что член BDE был признан членом трех групп. Поскольку силы, действующие на него, не параллельны, они должны быть параллельны.Мы видим это на рисунке ниже.

Следовательно, в этом случае точка D является точкой параллелизма.

Определив силу в точке B, теперь мы можем использовать диаграмму свободного тела балки ABC, чтобы найти неизвестные реакционные нагрузки в точке A.

Начнем с использования равновесия моментов с точкой А в качестве центра момента. Мы должны быть осторожны с единицами измерения из-за наличия парного момента в C с единицами Н · м.

Положительный знак, подтверждающий правильность принятого направления.Теперь, суммируя силы в направлениях x и y, мы решаем неизвестные силы реакции в точке A.

=>

При отсутствии другой силы в направлении x RAx равен нулю.